NR293 Final Exam Review & Practice Questions

Lakukan tugas rumah & ujian kamu dengan baik sekarang menggunakan Quizwiz!

Alendronate (Fosamax)

- This medication is used to prevent and treat postmenopausal osteoporosis - must be taken first thing in the morning on an empty stomach. - The pt must be instructed to wait at least 30 minutes after taking Alendronate before eating, drinking, or taking other medications.

Amphotericin B: nursing implications

- To reduce the severity of the infusion-related reactions, pretreatment with an antipyretic (acetaminophen), antihistamines, antiemetics, and corticosteroids may be given. - Use IV infusion pumps and the most distal veins possible.

Tuberculosis treatment

- Total treatment is approximately 9 months - reinforce medication regime and importance of compliance

Benzodiazepines adverse effects

- Drowsiness - Fall hazard for elderly persons - "Hangover" effect/daytime sleepiness

alendronate adverse effects

- Risk of esophageal burns if medication lodges in esophagus before reaching the stomach - Risk of osteonecrosis of the jaw - Possible severe (incapacitating) bone, joint, or muscle pain

Zolpidem (Ambien)

- Sedative/Hypnotic - used for sleep - Somnambulation - Lower incidence of daytime sleepiness compared with benzodiazepine hypnotics

Osmotic Diuretics: Indications

- Treatment of clients in the early, oliguric phase of acute renal failure (ARF) - To reduce intracranial pressure - Treatment of cerebral edema - NOT indicated for peripheral edema

oseltamivir patient teaching

- Treatment should begin within 2 days of symptoms onset - Instruct on the correct usage - Encourage the patient to complete the full course

What is ganciclovir used to treat?

used to treat infection with CMV

Orlistat (Xenical)

- CNS Stimulant - Is prescribed to morbidly obese pts to lose weight. It prevents absorption of some of the fat in the pts diet. - The med should be taken 3 times/day within 1 hour of a meal - Its a lipase inhibitor, not an appetite suppressor.

Opioid adverse effects

- CNS depression - Leads to respiratory depression - Nausea and vomiting - Urinary retention - Diaphoresis and flushing - Pupil constriction (miosis) - Constipation - Itching

General side effects of muscle relaxants

***drowsiness*** muscle weakness euphoria lightheadedness

What are CNS stimulants used for?

- ADHD, ADD - Narcolepsy - Weight loss - Migraines

Clozapine (Clozaril)

- Antipsychotic medication - Given to bipolar patients to offset psychotic behavior - Can cause tachycardia, weight gain, blood dyscrasias

effects of chemo drugs on the body

- Bone marrow suppression- neutropenia, thrombocytopenia; filgrastim given for neutropenia and oprelvekin give for filgrastim given for neutropenia and oprelvekin give for thrombocytopeniathrombocytopenia - Severe nausea and vomiting- Imatinib; medicate with antiemetic Severe nausea and vomiting- Imatinib; medicate with antiemetic 60 minutes prior to dose60 minutes prior to dose - Hyperuricemia- Hydroxyurea; give allopurinol - Octreotide given to less carcinoid crisis (severe diarrhea and Octreotide given to less carcinoid crisis (severe diarrhea and potential life-threatening hypotension)potential life-threatening hypotension) - Teratogenic - Acute left ventricular failure - Pulmonary Fibrosis and Pneumonitis - Renal toxicity - Heart failure

Magnesium Salts

- Commonly cause diarrhea - Dangerous when used with renal failure

levothyrozine (Synthroid) patient education

- During pregnancy, treatment for hypothyroidism should continue - Teach patient to take thyroid drugs once daily in the morning to decrease the likelihood of insomnia if taken later in the day - Teach patient to take the medications at the same time every day and not to switch brands without primary care provider approval - Teach patient to take thyroid drugs on an empty stomach with no other medications - Teach patients to report any unusual symptoms, chest pain, or heart palpitations - Teach patients not to take over-the-counter medications without primary care provider approval - Teach patients that therapeutic effects may take several weeks to occur - Avoid eating foods high in iodine (seafood, soy sauce, tofu, and iodized salt) - Therapy for hypothyroidism is life long - TSH level is 0.5-5.0

Cholinergics patient teaching

- Encourage clients with myasthenia gravis to take medication 30 minutes before eating to help improve chewing and swallowing - When cholinergic drugs are prescribed for Alzheimer's disease, be honest with caregivers and clients that the drugs are for management of symptoms (not a cure) - Therapeutic effects of anti-Alzheimer's drugs may not occur for up to 6 weeks

Warfarin (Coumadin)

- Given orally only - Monitored by prothrombin time and INR (PT-INR) - Many dietary considerations

Aluminum salts

- Have constipating effects - Often used with magnesium to counteract constipation - Often recommended for patients with renal diseas

Patient teaching for BPH drugs (Tamsulosin)

- Instruct clients that these medications should never be stopped abruptly - Teach clients to change positions slowly to prevent or minimize postural hypotension - Rebound hypertension or chest pain may occur if this medication is discontinued abruptly - Inform clients to report the following to their physician (s/s of heart failure)

antiepileptic drugs: General teaching

- Instruct clients to wear a medical alert tag or ID - Antiseizure medications should not be discontinued abruptly- can cause withdrawal seizures - Follow driving recommendations - Teach clients that therapy is long term and possibly lifelong (not a cure)

Osmotic Diuretics: Mannitol

- Intravenous infusion only - May crystallize when exposed to low temperatures - Use of a filter is required

what are extrapyramidal symptoms (EPS)?

- Involuntary muscle symptoms similar to those of Parkinson's disease - Akathisia (distressing muscle restlessness) - Acute dystonia (painful muscle spasms)

Enoxaparin (Lovenox)

- LMWH (Low molecular weight heparin) - Given subcutaneously in abdomen only - requires plt monitoring

What do we monitor for in patients on aminoglycosides?

- Monitor hearing, s/s tinnitus, dizziness, vertigo - Monitor I & O's, BUN/Cr, daily weights

Cephalosporins adverse reactions

- Nephrotoxicity - Antabuse effect/disulfiram-like reaction- occurs if alcohol is consumed within 72 hours and causes flushing, throbbing in head, respiratory distress, sweating, chest pain, and hypotension- can lead to arrhythmias and unconsciousness

Opioid considerations

- Oral forms should be taken with food to minimize gastric upset - Ensure safety measures, such as keeping side rails up, to prevent injury - Constipation is a common adverse effect and may be prevented with adequate fluid and fiber intake- may need stool softener

Aminoglycosides peak and trough

- Peak: highest drug levels for once-daily regimens - Trough: lowest to ensure adequate renal clearance of the drug and avoid toxicity

Loop diuretics drug effects

- Potent diuresis and subsequent loss of fluid - Potassium and Sodium depletion (especially potassium) - Decreased blood pressure

Amphotericin B adverse effect

- Renal toxicity - Fever, chills, headache, nausea, occasional hypotension

Vasopressin and Desmopressin

- Used in the treatment of diabetes insipidus - Because of their vasoconstrictor properties, they are useful in the treatment of various types of bleeding, in particular gastrointestinal hemorrhage - Desmopressin is useful in the treatment of hemophilia A and type I von Willebrand's disease because of its effects on various blood-clotting factors - should reduce severe thirst and decrease urinary output

MAOIs patient teaching

- avoid foods containing tyramine - may take 2-4 weeks to take effects - causes agitation, CNS stimulation - get up slowly - be very careful with other meds

What can long-term therapy with phenytoin cause?

- gingival hyperplasia - bone marrow suppression - acne - hirsutism - Dilantin facies (short nose, flat face, large head)

BPH drug (Tamsulosin) adverse effects

- headache - abnormal ejaculation - rhinitis

Calcium Salts

- may cause constipation and kidney stones - Often advertised as an extra source of dietary calcium

Cyclobenzaprine patient teaching

- may cause drowsiness, dizziness, and blurred vision, do not take with alcohol or other CNS depressants - do not drive - increase fluid intake if constipation occurs - notify provider of urinary retention - notify provider immediately of signs and symptoms of serotonin syndrome - perform good hygiene

oral iron patient education

- may stain teeth so use a straw - Keep away from children because oral forms may look like candy - should be taken on an empty stomach for maximum absorption - should be given with juices high in vitamin c to enhance absorption, but not with milk or antacids - remain upright for 15 to 30 minutes after oral iron doses to avoid esophageal corrosion

Cyclosporine: Patient Teaching

- mix oral cyclosporine in a glass container - Do not use styrofoam because the drug adheres to the inside wall of the container. - take with food to minimize GI upset - no grapefruit juice - therapy for organ transplantation is life long

What is metoclopramide used for?

- nausea and vomiting - stimulate peristalsis in GI tract, enhancing emptying of stomach contents - GERD & delayed gastric emptying

SSRI patient teaching

- the time period for SSRIs to take effect is 4 to 6 weeks - can cause GI upset, and taking them with food is recommended - should be taken in the morning to prevent any sleep disturbances.

Antihistamines common properties

-Antihistaminic -Anticholinergic -Sedative

Antihistamines

-Have anticholinergic effects, making them more effective than non-sedating drugs in some cases- they dry you up - cause drowsiness - Should be given early in treatment before all the histamine binds to the receptors - take with meals

Antibiotics: Patient Teaching

-Instruct patient to take entire prescribed product AROUND THE CLOCK even if they start to feel better -report any signs of allergic reaction or superinfection -teach that antibiotics CAN interfere with oral contraceptives (use another form of birth control) -avoid alcohol w/ cephalosporin -> alcohol intolerance -> abdominal cramping, N, V, headache, decreased BP, shortness of breath, cardiac palpitations, pulse rate greatly increases, sweating, and flushing. -use sunscreen with tetracycline -> avoid prolonged exposure to sun -> wear protective clothing/sunglasses (no tanning salon -dont take with milk or milk products or iron or magnesium containing laxatives and dont take antacids

Benzodiazepines indications

-Reduce anxiety -Induce sedation, relax muscles, inhibit convulsions -Treat alcohol and drug withdrawal symptoms -Safer than sedative-hypnotics

patient teaching for benzodiazepines

-Take as prescribed, don't abruptly stop or change dose without instruction from doctor -When discontinuing, drug must be tapered over several weeks -Avoid alcohol and other CNS depressants -Advise the client and family to watch for manifestations of overdose. Notify the provider if these occur -Medications should be taken with, or shortly after, meals or snacks to reduce gastrointestinal discomfort

Ethambutol adverse reaction

Antitubercular med adverse reaction: - optic neuritis (decrease in visual acuity and color perception)

Beta Blocker Assessment

Apical pulse for 1 minute

What do benzodiazepines end in?

-lam or -pam

What do beta blockers end in?

-lol

normal therapeutic drug level of digoxin

0.5-2 ng/mL

when should antacids be administered?

1 to 2 hours before meds

beta blockers patient teaching

1. Abrupt discontinuation of beta blockers can lead to rebound hypertension. 2. discuss OTC meds with RN, pharmacist or prescriber, as many of them are contraindicated for patient with hypertension. 3. Advise patients of possibility of orthostatic hypotension. 4. advise patient to report dizziness, slow pulse rate, to health care provider.

Phenytoin therapeutic drug levels

10 to 20 mcg/mL

Thiazides should not be used if creatinine clearance is less than __________mL/min (normal is 125 mL/min).

30 to 50

When should antiemetics be administered before chemo?

30-60 minutes

What is the max dose of Ibuprofen in a day?

3200 mg/day

Heparin aPTT therapeutic range

50-80 seconds

The nurse is providing patient education for a patient taking an oral contraceptive. Which drugs may cause interactions with oral contraceptives? (Select all that apply.) a. Cephalexin (Keflex) b. Guaifenesin (Robitussin) c. Warfarin (Coumadin) d. Ibuprofen (Motrin) e. Theophylline (Uniphyl)

A, C, E a. Cephalexin (Keflex) c. Warfarin (Coumadin) e. Theophylline (Uniphyl) Patients must be educated about the need to use alternative birth control methods for at least 1 month during and after taking any of these drugs: antibiotics (especially penicillins and cephalosporins); barbiturates; isoniazid; and rifampin. The effectiveness of other drugs, such as anticonvulsants, beta blockers, hypnotics, antidiabetic drugs, warfarin, theophylline, tricyclic antidepressants, and vitamins, may be reduced when they are taken with oral contraceptives.

what does methylphenidate hydrochloride treat?

ADHD (can also be used for narcolepsy)

A patient is experiencing status epilepticus. The nurse prepares to give which drug of choice for the treatment of this condition? a. Diazepam (Valium) b. Midazolam (Versed) c. Valproic acid (Depakote) d. Carbamazepine (Tegretol)

ANS: A Diazepam (Valium) is considered by many to be the drug of choice for status epilepticus.

The nurse is teaching a group of patients about self-administration of insulin. What content is important to include? a. Patients need to use the injection site that is the most accessible. b. If two different insulins are ordered, they need to be given in separate injections. c. When mixing insulins, the cloudy (such as NPH) insulin is drawn up into the syringe first. d. When mixing insulins, the clear (such as regular) insulin is drawn up into the syringe first.

ANS: D If mixing insulins in one syringe, the clear (regular) insulin is always drawn up into the syringe first. Patients always need to rotate injection sites. Mixing of insulins may be ordered.

The nurse notes in the patient's medication history that the patient is taking cyclobenzaprine (Flexeril). Based on this finding, the nurse interprets that the patient has which disorder? a. A musculoskeletal injury b. Insomnia c. Epilepsy d. Agitation

ANS: A Cyclobenzaprine (Flexeril) is the muscle relaxant most commonly used to reduce spasms following musculoskeletal injuries. It is not appropriate for insomnia, epilepsy, or agitation.

When reviewing the medication orders for a patient who is taking penicillin, the nurse notes that the patient is also taking the oral anticoagulant warfarin (Coumadin). What possible effect may occur as the result of an interaction between these drugs? a. The penicillin will cause an enhanced anticoagulant effect of the warfarin. b. The penicillin will cause the anticoagulant effect of the warfarin to decrease. c. The warfarin will reduce the anti-infective action of the penicillin. d. The warfarin will increase the effectiveness of the penicillin.

ANS: A Administering penicillin reduces the vitamin K in the gut (intestines); therefore, enhanced anticoagulant effect of warfarin may occur.

A patient is asking advice about which over-the-counter antacid is considered the most safe to use for heartburn. The nurse explains that the reason that calcium antacids are not used as frequently as other antacids is for which of these reasons? a. Their use may result in kidney stones. b. They cause decreased gastric acid production. c. They cause severe diarrhea. d. Their use may result in fluid retention and edema.

ANS: A Calcium antacids are not used as frequently as other antacids because their use may lead to the development of kidney stones; they also cause increased gastric acid production.

A 22-year-old patient has been taking lithium for 1 year, and the most recent lithium level is 0.9 mEq/L. Which statement about the laboratory result is correct? a. The lithium level is therapeutic. b. The lithium level is too low. c. The lithium level is too high. d. Lithium is not usually monitored with blood levels.

ANS: A Desirable long-term maintenance lithium levels range between 0.6 and 1.2 mEq/L.

Which action is most appropriate regarding the nurse's administration of a rapid-acting insulin to a hospitalized patient? a. Give it within 15 minutes of mealtime. b. Give it after the meal has been completed. c. Administer it once daily at the time of the midday meal. d. Administer it with a snack before bedtime.

ANS: A Rapid-acting insulins, such as insulin lispro and insulin aspart, are able to mimic closely the body's natural rapid insulin output after eating a meal; for this reason, both insulins are usually administered within 15 minutes of the patient's mealtime.

When monitoring a patient who is taking hydrochlorothiazide (HydroDIURIL), the nurse notes that which drug is most likely to cause a severe interaction with the diuretic? a. Digitalis b. Penicillin c. Potassium supplements d. Aspirin

ANS: A There is an increased risk for digitalis toxicity in the presence of hypokalemia, which may develop with hydrochlorothiazide therapy. Potassium supplements are often prescribed with hydrochlorothiazide therapy to prevent hypokalemia. The other options do not have interactions with hydrochlorothiazide.

The nurse is teaching a group of patients about management of diabetes. Which statement about basal dosing is correct? a. "Basal dosing delivers a constant dose of insulin." b. "With basal dosing, you can eat what you want and then give yourself a dose of insulin." c. "Glargine insulin is given as a bolus with meals." d. "Basal-bolus dosing is the traditional method of managing blood glucose levels."

ANS: A Basal-bolus therapy is the attempt to mimic a healthy pancreas by delivering basal insulin constantly as a basal, and then as needed as a bolus. Glargine insulin is used as a basal dose, not as a bolus with meals. Basal-bolus therapy is a newer therapy; historically, sliding-scale coverage was implemented.

A patient who has received some traumatic news is panicking and asks for some medication to help settle down. The nurse anticipates giving which drug that is most appropriate for this situation? a. Diazepam (Valium) b. Zolpidem (Ambien) c. Phenobarbital d. Cyclobenzaprine (Flexeril)

ANS: A Benzodiazepines such as diazepam are used as anxiolytics, or sedatives. Zolpidem is used as a hypnotic for sleep. Phenobarbital is not used as an anxiolytic but is used for seizure control. Cyclobenzaprine is a muscle relaxant and is not used to reduce anxiety.

During a teaching session about self-monitoring while taking a beta blocker at home, the nurse has taught the patient to take his apical pulse daily for 1 minute. If the pulse rate decreases to less than 60 beats/min, the nurse will instruct the patient to: a. notify his prescriber. b. reduce the dose of his beta blocker by half. c. continue the medication because this is an expected effect. d. skip the medication dose that day, and check his pulse again the next day.

ANS: A Cardiac depression can occur with beta blockers; instruct the patient to contact his prescriber if his pulse rate decreases to less than 60 per minute. The medication dose may need to be changed, but it is not appropriate for the nurse to change the dosage.

The nurse is reviewing the sputum culture results of a patient with pneumonia and notes that the patient has a gram-positive infection. Which generation of cephalosporin is most appropriate for this type of infection? a. First generation b. Second generation c. Third generation d. Fourth generation

ANS: A First-generation cephalosporins provide excellent coverage against gram-positive bacteria but limited coverage against gram-negative bacteria.

Phenytoin (Dilantin) has a narrow therapeutic index. The nurse recognizes that this characteristic indicates which of these? a. The safe and the toxic plasma levels of the drug are very close to each other. b. The phenytoin has a low chance of being effective. c. There is no difference between safe and toxic plasma levels. d. A very small dosage can result in the desired therapeutic effect.

ANS: A Having a "narrow therapeutic index" means that there is a small difference between safe and toxic drug levels. These drugs require monitoring of therapeutic plasma levels.

A patient, newly diagnosed with hypothyroidism, has received a prescription for thyroid replacement therapy. The nurse will instruct the patient to take this medication at which time of day? a. In the morning b. With the noon meal c. With the evening meal d. At bedtime

ANS: A If possible, it is best to administer thyroid drugs taken once daily in the morning so as to decrease the likelihood of insomnia that may result from evening dosing.

The U.S. Food and Drug Administration has issued a warning for users of antiepileptic drugs. Based on this report, the nurse will monitor for which potential problems with this class of drugs? a. Increased risk of suicidal thoughts and behaviors b. Signs of bone marrow depression c. Indications of drug addiction and dependency d. Increased risk of cardiovascular events, such as strokes

ANS: A In December 2008, the U.S. Food and Drug Administration (FDA) required black box warnings on all antiepileptic drugs regarding the risk of suicidal thoughts and behaviors. Patients being treated with antiepileptic drugs for any indication need to be monitored for the emergence or worsening of depression, suicidal thoughts or behavior, or any unusual changes in mood or behavior.

The nurse is administering intravenous acyclovir (Zovirax) to a patient with a viral infection. Which administration technique is correct? a. Infuse intravenous acyclovir slowly, over at least 1 hour. b. Infuse intravenous acyclovir by rapid bolus. c. Refrigerate intravenous acyclovir. d. Restrict oral fluids during intravenous acyclovir therapy.

ANS: A Intravenous acyclovir is stable for 12 hours at room temperature and often precipitates when refrigerated. Intravenous infusions must be diluted as recommended (e.g., with 5% dextrose in water or normal saline) and infused with caution. Infusion over longer than 1 hour is suggested to avoid the renal tubular damage seen with more rapid infusions. Adequate hydration should be encouraged (unless contraindicated) during the infusion and for several hours afterward to prevent drug-related crystalluria.

A patient is being discharged to home on a single daily dose of a diuretic. The nurse instructs the patient to take the dose at which time so it will be least disruptive to the patient's daily routine? a. In the morning b. At noon c. With supper d. At bedtime

ANS: A It is better to take the diuretic medication early in the morning to prevent urination during the night. Taking the diuretic at the other times may cause nighttime urination and disrupt sleep.

Mannitol (Osmitrol) has been ordered for a patient with acute renal failure. The nurse will administer this drug using which procedure? a. Intravenously, through a filter b. By rapid intravenous bolus c. By mouth in a single morning dose d. Through a gravity intravenous drip with standard tubing

ANS: A Mannitol is administered via intravenous infusion through a filter because of possible crystallization. It is not available in oral form.

A patient is recovering from a minor automobile accident that occurred 1 week ago. He is taking cyclobenzaprine (Flexeril) for muscular pain and goes to physical therapy three times a week. Which nursing diagnosis would be appropriate for him? a. Risk for injury related to decreased sensorium b. Risk for addiction related to psychologic dependency c. Decreased fluid volume related to potential adverse effects d. Disturbed sleep pattern related to the drug's interference with REM sleep

ANS: A Musculoskeletal relaxants have a depressant effect on the CNS; thus, the patient needs to be taught the importance of taking measures to minimize self-injury and falls related to decreased sensorium. "Risk for addiction" is not a NANDA nursing diagnosis. The other nursing diagnoses are not appropriate for this situation

A patient is taking flurazepam (Dalmane) three to four nights a week for sleeplessness. She is concerned that she cannot get to sleep without taking the medication. What nonpharmacologic measures should the nurse suggest to promote sleep for this patient? a. Providing a quiet environment b. Exercising before bedtime to become tired c. Consuming heavy meals in the evening to promote sleepiness d. Drinking hot tea or coffee just before bedtime

ANS: A Nonpharmacologic approaches to induce sleep include providing a quiet environment, avoiding heavy exercise before bedtime, avoiding heavy meals late in the evening, and drinking warm decaffeinated drinks, such as warm milk, before bedtime.

During an intravenous (IV) infusion of amphotericin B, a patient develops tingling and numbness in his toes and fingers. What will the nurse do first? a. Discontinue the infusion immediately. b. Reduce the infusion rate gradually until the adverse effects subside. c. Administer the medication by rapid IV infusion to reduce these effects. d. Nothing; these are expected side effects of this medication.

ANS: A Once the intravenous infusion of amphotericin B has begun, vital signs must be monitored frequently to assess for adverse reactions such as cardiac dysrhythmias, visual disturbances, paresthesias (numbness or tingling of the hands or feet), respiratory difficulty, pain, fever, chills, and nausea. If these adverse effects or a severe reaction occur, the infusion must be discontinued (while the patient is closely monitored) and the prescriber contacted. The other options are incorrect.

When administering digoxin immune Fab (Digibind) to a patient with severe digoxin toxicity, the nurse knows that each vial can bind with how much digoxin? a. 0.5 mg b. 5 mg c. 5.5 mg d. 15 mg

ANS: A One vial of digoxin immune Fab binds 0.5 mg of digoxin.

A patient is taking an alpha blocker as treatment for benign prostatic hyperplasia. The nurse will monitor for which potential drug effect? a. Orthostatic hypotension b. Increased blood pressure c. Decreased urine flow d. Discolored urine

ANS: A Orthostatic hypotension can occur with any dose of an alpha blocker, and patients must be warned to get up slowly from a supine position. The other responses are not drug effects of alpha blockers.

A patient is going home with a new prescription for the beta-blocker atenolol (Tenormin). The nurse will include which content when teaching the patient about this drug? a. Never stop taking this medication abruptly. b. The medication will be stopped once symptoms subside. c. If adverse effects occur, stop taking the drug for 24 hours, and then resume. d. Be watchful for first-dose hypotension.

ANS: A Patients need to be weaned off these medications slowly because rebound hypertension and chest pain are possible with abrupt withdrawal. The drugs should never be stopped abruptly nor doses skipped. First-dose hypotension occurs with alpha blockers.

A patient is receiving a tube feeding through a gastrostomy. The nurse expects that which type of drug will be used to promote gastric emptying for this patient? a. Prokinetic drugs, such as metoclopramide (Reglan) b. Serotonin blockers, such as ondansetron (Zofran) c. Anticholinergic drugs, such as scopolamine (Transderm-Scop) d. Neuroleptic drugs, such as chlorpromazine (Thorazine)

ANS: A Prokinetic drugs promote the movement of substances through the gastrointestinal tract and increase gastrointestinal motility.

The nurse has given medication instructions to a patient receiving phenytoin (Dilantin). Which statement by the patient indicates that the patient has an adequate understanding of the instructions? a. "I will need to take extra care of my teeth and gums while on this medication." b. "I can go out for a beer while on this medication." c. "I can skip doses if the side effects bother me." d. "I will be able to stop taking this drug once the seizures stop."

ANS: A Scrupulous dental care is necessary to prevent gingival hypertrophy during therapy with phenytoin. Alcohol and other central nervous system depressants may cause severe sedation. Consistent dosing is important to maintain therapeutic drug levels. Therapy with AEDs usually must continue for life and must not be stopped once seizures stop.

A patient is taking digoxin (Lanoxin) and a loop diuretic daily. When the nurse enters the room with the morning medications, the patient states, "I am seeing a funny yellow color around the lights." What is the nurse's next action? a. Assess the patient for symptoms of digoxin toxicity. b. Withhold the next dose of the diuretic. c. Administer the digoxin and diuretic together as ordered. d. Document this finding, and reassess in 1 hour.

ANS: A Seeing colors around lights is one potential indication of developing digoxin toxicity. If a patient complains of this, the nurse needs to assess for other signs and symptoms of digoxin toxicity including bradycardia, headache, dizziness, confusion, nausea, and blurred vision, and then notify the prescriber. Administering the drug or withholding the diuretic are incorrect options.

A patient in the intensive care unit has a nasogastric tube and is also receiving a proton pump inhibitor (PPI). The nurse recognizes that the purpose of the PPI is which effect? a. Prevent stress ulcers b. Reduce bacteria levels in the stomach c. Reduce gastric gas formation (flatulence) d. Promote gastric motility

ANS: A Stress-related mucosal damage is an important issue for critically ill patients. Stress ulcer prophylaxis (or therapy to prevent severe gastrointestinal [GI] damage) is undertaken in almost every critically ill patient in an intensive care unit and for many patients on general medical surgical units. Procedures performed commonly in critically ill patients, such as passing nasogastric tubes, placing patients on ventilators, and others, predispose patients to bleeding of the GI tract. Guidelines suggest that all such patients receive either a histamine receptor-blocking drug or a proton pump inhibitor.

During therapy with amphotericin B, the nurse will monitor the patient for known adverse effects that would be reflected by which laboratory result? a. Serum potassium level of 2.7 mEq/L b. Serum potassium level of 5.8 mEq/L c. White blood cell count of 7000 cells/mm3 d. Platelet count of 300,000/ microliter

ANS: A The nurse needs to monitor for hypokalemia, a possible adverse effect of amphotericin B.

The nurse is administering insulin lispro (Humalog) and will keep in mind that this insulin will start to have an effect within which time frame? a. 15 minutes b. 1 to 2 hours c. 80 minutes d. 3 to 5 hours

ANS: A The onset of action for insulin lispro is 15 minutes. The peak plasma concentration is 1 to 2 hours; the elimination half-life is 80 minutes; and the duration of action is 3 to 5 hours.

A patient who has severe nausea and vomiting following a case of food poisoning comes to the urgent care center. When reviewing his medication history, the nurse notes that he has an allergy to procaine. The nurse would question an order for which antiemetic drug if ordered for this patient? a. Metoclopramide (Reglan) b. Promethazine (Phenergan) c. Phosphorated carbohydrate solution (Emetrol) d. Palonosetron (Aloxi)

ANS: A The use of metoclopramide (Reglan) is contraindicated in patients with a hypersensitivity to procaine or procainamide. There are no known interactions with the drugs listed in the other options.

A patient with gout has been treated with allopurinol (Zyloprim) for 2 months. The nurse will monitor laboratory results for which therapeutic effect? a. Decreased uric acid levels b. Decreased prothrombin time c. Decreased white blood cell count d. Increased hemoglobin and hematocrit levels

ANS: A Treatment of gout with allopurinol should result in decreased uric acid levels. The other options are incorrect.

Which drug classes are considered first-line treatment for heart failure? (Select all that apply.) a. Angiotensin-converting enzyme (ACE) inhibitors b. Angiotensin II receptor blockers (ARBs) c. Digoxin (cardiac glycoside) d. Beta blockers e. Nesiritide (Natrecor), the B-type natriuretic peptide

ANS: A, B, D a. Angiotensin-converting enzyme (ACE) inhibitors b. Angiotensin II receptor blockers (ARBs) d. Beta blockers ACE inhibitors, ARBs, and beta blockers are now considered the first-line treatments for heart failure. Digoxin is used when the first-line treatments are not successful; nesiritide is considered a last-resort treatment.

A beta blocker is prescribed for a patient with angina. The nurse reviews the orders for other drugs that may interact with the beta blocker. Which drugs or drug classes are known to have an interaction with a beta blocker? (Select all that apply.) a. Diuretics b. Anticholinergics c. Penicillins d. Oral hypoglycemics e. Alcohol f. Anticoagulants

ANS: A, B, D, E a. Diuretics b. Anticholinergics d. Oral hypoglycemics e. Alcohol When taken with beta blockers, diuretics and alcohol may cause additive hypotensive effects; oral hypoglycemic medications may cause the blood glucose to decrease; and anticholinergics may cause decreased beta-blocker effects. Penicillins and anticoagulants are not known to interact with beta blockers.

Levothyroxine (Synthroid) has been prescribed for a patient with hypothyroidism. The nurse reviews the patient's current medications for potential interactions. Which of these drugs or drug classes interact with levothyroxine? (Select all that apply.) a. Phenytoin (Dilantin) b. Estrogens c. Beta blockers d. Warfarin (Coumadin) e. Penicillins f. Iron supplements

ANS: A, B, D, F a. Phenytoin (Dilantin) b. Estrogens d. Warfarin (Coumadin) f. Iron supplements Drug interactions with thyroid preparations include phenytoin, cholestyramine, antacids, calcium salts, iron products, estrogens, and warfarin.

When assessing the medication history of a patient with a new diagnosis of Parkinson's disease, which conditions are contraindications for the patient who will be taking carbidopa-levodopa? (Select all that apply.) a. Angle-closure glaucoma b. History of malignant melanoma c. Hypertension d. Benign prostatic hyperplasia e. Concurrent use of monoamine oxidase inhibitors (MAOIs)

ANS: A, B, E a. Angle-closure glaucoma b. History of malignant melanoma e. Concurrent use of monoamine oxidase inhibitors (MAOIs) Angle-closure glaucoma, a history of melanoma or other undiagnosed skin conditions, and concurrent use of MAOIs are contraindications to the use of carbidopa-levodopa. The other options are incorrect.

The nurse is reviewing the history of a patient who has a new order for a nonsteroidal antiinflammatory drug (NSAID) to treat tendonitis. Which conditions are contraindications to the use of NSAIDs? (Select all that apply.) a. Vitamin K deficiency b. Arthralgia c. Peptic ulcer disease d. Neuropathy e. Pericarditis

ANS: A, C a. Vitamin K deficiency c. Peptic ulcer disease Contraindications to NSAIDs include known drug allergy as well as conditions that place the patient at risk for bleeding, such as Vitamin K deficiency and peptic ulcer disease. NSAIDs may be used to treat arthralgia and pericarditis. Neuropathy is not a contraindication.

Methotrexate is ordered for a patient with a malignant tumor, and the nurse is providing education about self-care after the chemotherapy is given. Which statements by the nurse are appropriate for the patient receiving methotrexate? (Select all that apply.) a. Report unusual bleeding or bruising. b. Hair loss is not expected with this drug. c. Prepare for hair loss. d. Avoid areas with large crowds or gatherings. e. Avoid foods that are too hot or too cold or rough in texture. f. Restrict fluid intake to reduce nausea and vomiting.

ANS: A, C, D, E a. Report unusual bleeding or bruising. c. Prepare for hair loss. d. Avoid areas with large crowds or gatherings. e. Avoid foods that are too hot or too cold or rough in texture. Counsel patients who are taking methotrexate to expect hair loss and to report any unusual bleeding or bruising. Because of the possibility of infection, avoid areas with large crowds or gatherings. Foods that are too hot or too cold or rough in texture may be irritating to the oral mucosa. Fluid intake is to be encouraged to prevent dehydration.

During antibiotic therapy, the nurse will monitor closely for signs and symptoms of a hypersensitivity reaction. Which of these assessment findings may be an indication of a hypersensitivity reaction? (Select all that apply.) a. Wheezing b. Diarrhea c. Shortness of breath d. Swelling of the tongue e. Itching f. Black, hairy tongue

ANS: A, C, D, E a. Wheezing c. Shortness of breath d. Swelling of the tongue e. Itching Hypersensitivity reactions may be manifested by wheezing; shortness of breath; swelling of the face, tongue, or hands; itching; or rash.

The nurse is administering intravenous vancomycin (Vancocin) to a patient who has had gastrointestinal surgery. Which nursing measures are appropriate? (Select all that apply.) a. Monitoring serum creatinine levels b. Restricting fluids while the patient is on this medication c. Warning the patient that a flushed feeling or facial itching may occur d. Instructing the patient to report dizziness or a feeling of fullness in the ears e. Reporting a trough drug level of 11 mcg/mL and holding the drug f. Reporting a trough drug level of 24 mcg/mL and holding the drug

ANS: A, C, D, F a. Monitoring serum creatinine levels c. Warning the patient that a flushed feeling or facial itching may occur d. Instructing the patient to report dizziness or a feeling of fullness in the ears f. Reporting a trough drug level of 24 mcg/mL and holding the drug Constant monitoring for drug-related neurotoxicity, nephrotoxicity, ototoxicity, and superinfection remain critical to patient safety. Monitor for nephrotoxicity by monitoring serum creatinine levels. Ototoxicity may be indicated if the patient experiences dizziness or a feeling of fullness in the ears, and these symptoms must be reported immediately. Vancomycin infusions may cause red man syndrome, which is characterized by flushing of the neck and face and a decrease in blood pressure. In addition, adequate hydration (at least 2 L of fluids every 24 hours unless contraindicated) is most important to prevent nephrotoxicity. Optimal trough blood levels of vancomycin are 10 to 20 mcg/mL; therefore, the drug should not be administered when there is a trough level of 24 mcg/mL.

A patient who has been taking a selective serotonin reuptake inhibitor (SSRI) is complaining of "feeling so badly" when he started taking an over-the-counter St. John's wort herbal product at home. The nurse suspects that he is experiencing serotonin syndrome. Which of these are symptoms of serotonin syndrome? (Select all that apply.) a. Agitation b. Drowsiness c. Tremors d. Bradycardia e. Sweating f. Constipation

ANS: A, C, E a. Agitation c. Tremors e. Sweating Common symptoms of serotonin syndrome include delirium, agitation, tachycardia, sweating, hyperreflexia, shivering, coarse tremors, and others.

The nurse is presenting a substance-abuse lecture for teenage girls and is asked about "roofies." The nurse recognizes that this is the slang term for which substance? a. Cocaine b. Flunitrazepam c. Secobarbital d. Methamphetamine

ANS: B Flunitrazepam is a benzodiazepine that has recently gained popularity as a recreational drug and is commonly called roofies (the "date-rape" drug). The other drugs are not known as roofies.

The nurse is providing patient education for a patient taking an oral contraceptive. Which drugs may cause interactions with oral contraceptives? (Select all that apply.) a. Cephalexin (Keflex) b. Guaifenesin (Robitussin) c. Warfarin (Coumadin) d. Ibuprofen (Motrin) e. Theophylline (Uniphyl)

ANS: A, C, E a. Cephalexin (Keflex) c. Warfarin (Coumadin) e. Theophylline (Uniphyl) Patients must be educated about the need to use alternative birth control methods for at least 1 month during and after taking any of these drugs: antibiotics (especially penicillins and cephalosporins); barbiturates; isoniazid; and rifampin. The effectiveness of other drugs, such as anticonvulsants, beta blockers, hypnotics, antidiabetic drugs, warfarin, theophylline, tricyclic antidepressants, and vitamins, may be reduced when they are taken with oral contraceptives.

A patient who is diagnosed with genital herpes is taking topical acyclovir. The nurse will provide which teaching for this patient? (Select all that apply.) a. "Be sure to wash your hands thoroughly before and after applying this medicine." b. "Apply this ointment until the lesion stops hurting." c. "Use a clean glove when applying this ointment." d. "If your partner develops these lesions, then he can also use the medication." e. "You will need to avoid touching the area around your eyes." f. "You will have to practice abstinence when these lesions are active."

ANS: A, C, E, F a. "Be sure to wash your hands thoroughly before and after applying this medicine." c. "Use a clean glove when applying this ointment." e. "You will need to avoid touching the area around your eyes." f. "You will have to practice abstinence when these lesions are active." This medication needs to be applied as long as prescribed, and the medication needs to be applied with clean gloves. Prescriptions should not be shared; if the partner develops these lesions, the partner will have to be evaluated before medication is prescribed, if needed. Eye contact should be avoided. The presence of active genital herpes lesions requires sexual abstinence.

The nurse is reviewing the use of central nervous system stimulants. Which of these are indications for this class of drugs? (Select all that apply.) a. Narcolepsy b. Depression c. Panic attacks d. Neonatal apnea e. Attention deficit hyperactivity disorder (ADHD) f. Appetite suppression

ANS: A, D, E, F a. Narcolepsy d. Neonatal apnea e. Attention deficit hyperactivity disorder (ADHD) f. Appetite suppression Central nervous system stimulants can be used for narcolepsy, neonatal apnea, ADHD, and appetite suppression in the treatment of obesity. They are not used for depression and panic attacks.

A patient will be taking oral iron supplements at home. The nurse will include which statements in the teaching plan for this patient? (Select all that apply.) a. Take the iron tablets with meals. b. Take the iron tablets on an empty stomach 1 hour before meals. c. Take the iron tablets with an antacid to prevent heartburn. d. Drink 8 ounces of milk with each iron dose. e. Taking iron supplements with orange juice enhances iron absorption. f. Stools may become loose and light in color. g. Stools may become black and tarry. h. Tablets may be crushed to enhance iron absorption.

ANS: A, E, G a. Take the iron tablets with meals. e. Taking iron supplements with orange juice enhances iron absorption. g. Stools may become black and tarry. Iron tablets need to be taken with meals to reduce gastrointestinal distress, but antacids and milk interfere with absorption. Orange juice enhances the absorption of iron. Stools may become black and tarry in patients who are on iron supplements. Tablets need to be taken whole, not crushed, and the patient needs to be encouraged to eat foods high in iron.

A 10-year-old patient will be started on methylphenidate hydrochloride (Ritalin) therapy. The nurse will perform which essential baseline assessment before this drug is started? a. Eye examination b. Height and weight c. Liver function studies d. Hearing test

ANS: B Assessment of baseline height and weight is important before beginning Ritalin therapy because it may cause a temporary slowing of growth in prepubertal children. The other studies are not as essential at this time.

A patient is being discharged on anticoagulant therapy. The nurse will include in the patient education conversation that it is important to avoid herbal products that contain which substance? a. Valerian b. Ginkgo c. Soy d. Saw palmetto

ANS: B Capsicum pepper, feverfew, garlic, ginger, ginkgo, St. John's wort, and ginseng are some herbals that have potential interactions with anticoagulants, especially with warfarin.

A patient is concerned about the body changes that have resulted from long-term prednisone therapy for the treatment of asthma. Which effect of this drug therapy would be present to support the nursing diagnosis of disturbed body image? a. Weight loss b. Weight gain c. Pale skin color d. Hair loss

ANS: B Facial erythema, weight gain, hirsutism, and "moon face" (characteristic of Cushing's syndrome) are possible body changes that may occur with long-term prednisone therapy.

While monitoring a depressed patient who has just started SSRI antidepressant therapy, the nurse will observe for which problem during the early time frame of this therapy? a. Hypertensive crisis b. Self-injury or suicidal tendencies c. Extrapyramidal symptoms d. Loss of appetite

ANS: B In 2005, the U.S. Food and Drug Administration (FDA) issued special black-box warnings regarding the use of all classes of antidepressants in both adult and pediatric patient populations. Data from the FDA indicated a higher risk for suicide in patients receiving these medications. As a result, current recommendations for all patients receiving antidepressants include regular monitoring for signs of worsening depressive symptoms, especially when the medication is started or the dosage is changed.

A patient is recovering from abdominal surgery, which he had this morning. He is groggy but complaining of severe pain around his incision. What is the most important assessment data to consider before the nurse administers a dose of morphine sulfate to the patient? a. His pulse rate b. His respiratory rate c. The appearance of the incision d. The date of his last bowel movement

ANS: B One of the most serious adverse effects of opioids is respiratory depression. The nurse must assess the patient's respiratory rate before administering an opioid.

A 19-year-old student was diagnosed with hypothyroidism and has started thyroid replacement therapy with levothyroxine (Synthroid). After 1 week, she called the clinic to report that she does not feel better. Which response from the nurse is correct? a. "It will probably require surgery for a cure to happen." b. "The full therapeutic effects may not occur for 3 to 4 weeks." c. "Is it possible that you did not take your medication as instructed?" d. "Let's review your diet; it may be causing absorption problems."

ANS: B Patients need to understand that it may take up to 3 to 4 weeks to see the full therapeutic effects of thyroid drugs.

A patient has had recent mechanical heart valve surgery and is receiving anticoagulant therapy. While monitoring the patient's laboratory work, the nurse interprets that the patient's international normalized ratio (INR) level of 3 indicates that: a. the patient is not receiving enough warfarin to have a therapeutic effect. b. the patient's warfarin dose is at therapeutic levels. c. the patient's intravenous heparin dose is dangerously high. d. the patient's intravenous heparin dose is at therapeutic levels.

ANS: B A normal INR (without warfarin) is 1.0. A therapeutic INR for patients who have had mechanical heart valve surgery ranges from 2.5 to 3.5, with a middle value of 3.

A patient has been taking an AED for several years as part of his treatment for partial seizures. His wife has called because he ran out of medication this morning and wonders if he can go without it for a week until she has a chance to go to the drugstore. What is the nurse's best response? a. "He is taking another antiepileptic drug, so he can go without the medication for a week." b. "Stopping this medication abruptly may cause withdrawal seizures. A refill is needed right away." c. "He can temporarily increase the dosage of his other antiseizure medications until you get the refill." d. "He can stop all medications because he has been treated for several years now."

ANS: B Abrupt discontinuation of antiepileptic drugs can lead to withdrawal seizures. The other options are incorrect. The nurse cannot change the dose or stop the medication without a prescriber's order.

A patient has been taking iron supplements for anemia for 2 months. During a follow-up assessment, the nurse will observe for which therapeutic response? a. Decreased weight b. Increased activity tolerance c. Decreased palpitations d. Increased appetite

ANS: B Absence of fatigue, increased activity tolerance and well-being, and improved nutrition status are therapeutic responses to iron supplementation

The nurse notes in a patient's medication history that the patient is taking allopurinol (Zyloprim). Based on this finding, the nurse interprets that the patient has which disorder? a. Rheumatoid arthritis b. Gout c. Osteoarthritis d. Systemic lupus erythematosus

ANS: B Allopurinol is indicated for the treatment of gout but is not indicated for the other disorders listed.

The nurse is administering intravenous iron dextran for the first time to a patient with anemia. After giving a test dose, how long will the nurse wait before administering the remaining portion of the dose? a. 30 minutes b. 1 hour c. 6 hours d. 24 hours

ANS: B Although anaphylactic reactions usually occur within a few moments after the test dose, it is recommended that a period of at least 1 hour elapse before the remaining portion of the initial dose is given.

A patient is receiving an aluminum-containing antacid. The nurse will inform the patient to watch for which possible adverse effect? a. Diarrhea b. Constipation c. Nausea d. Abdominal cramping

ANS: B Aluminum-based antacids have a constipating effect as well as an acid-neutralizing capacity.

The nurse is reviewing the mechanism of action of antidiarrheal drugs. Which type of antidiarrheal medication works by decreasing the intestinal muscle tone and peristalsis of the intestines? a. Adsorbents such as Pepto-Bismol b. Anticholinergics such as belladonna alkaloids c. Probiotics such as Lactinex d. Lubricants such as mineral oil

ANS: B Anticholinergic drugs work to slow peristalsis by reducing the rhythmic contractions and the smooth muscle tone of the gastrointestinal tract.

When reviewing the laboratory values of a patient who is taking antithyroid drugs, the nurse will monitor for which adverse effect? a. Decreased glucose levels b. Decreased white blood cell count c. Increased red blood cell count d. Increased platelet count

ANS: B Antithyroid drugs may cause bone marrow suppression, resulting in agranulocytosis, leukopenia, thrombocytopenia, and other problems.

The nurse is providing patient teaching about the oral bisphosphonate alendronate (Fosamax). Which statement by the patient indicates a good understanding of when this drug should be taken? a. "I will take it in the evening just before bedtime." b. "I will take it in the morning with an 8-ounce glass of water." c. "I will take it with the first bite of the morning meal." d. "I will take it between meals on an empty stomach."

ANS: B Bisphosphonates must be taken in the morning, with 6 to 8 ounces of plain water, to prevent esophageal erosion. In addition, ***the patient must sit upright for 30 minutes after taking them.****

When reviewing the health history of a patient who will be receiving antacids, the nurse recalls that antacids containing magnesium need to be used cautiously in patients with which condition? a. Peptic ulcer disease b. Renal failure c. Hypertension d. Heart failure

ANS: B Both calcium- and magnesium-based antacids are more likely to accumulate to toxic levels in patients with renal disease and are commonly avoided in this patient group.

A patient with a history of chronic obstructive pulmonary disease (COPD) and type 2 diabetes has been treated for pneumonia for the past week. The patient has been receiving intravenous corticosteroids as well as antibiotics as part of his therapy. At this time, the pneumonia has resolved, but when monitoring the blood glucose levels, the nurse notices that the level is still elevated. What is the best explanation for this elevation? a. The antibiotics may cause an increase in glucose levels. b. The corticosteroids may cause an increase in glucose levels. c. His type 2 diabetes has converted to type 1. d. The hypoxia caused by the COPD causes an increased need for insulin.

ANS: B Corticosteroids can antagonize the hypoglycemic effects of insulin, resulting in elevated blood glucose levels.

The nurse notes in a patient's medication history that the patient has been taking desmopressin (DDAVP). Based on this finding, the nurse interprets that the patient has which disorder? a. Diabetes mellitus b. Diabetes insipidus c. Adrenocortical insufficiency d. Carcinoid tumor

ANS: B Desmopressin is used to prevent or control polydipsia (excessive thirst), polyuria, and dehydration in patients with diabetes insipidus. The symptoms are caused by a deficiency of endogenous antidiuretic hormone.

A patient on chemotherapy is using ondansetron (Zofran) for treatment of nausea. The nurse will instruct the patient to watch for which adverse effect of this drug? a. Dizziness b. Diarrhea c. Dry mouth d. Blurred vision

ANS: B Diarrhea is an adverse effect of the serotonin blockers. The other adverse effects listed may occur with anticholinergic drugs.

A patient who has been newly diagnosed with vertigo will be taking an antihistamine antiemetic drug. The nurse will include which information when teaching the patient about this drug? a. The patient may skip doses if the patient is feeling well. b. The patient will need to avoid driving because of possible drowsiness. c. The patient may experience occasional problems with taste. d. It is safe to take the medication with a glass of wine in the evening to help settle the stomach.

ANS: B Drowsiness may occur because of central nervous system (CNS) depression, and patients should avoid driving or working with heavy machinery because of possible sedation. These drugs must not be taken with alcohol or other CNS depressants because of possible additive depressant effects. The medication should be taken as instructed and not skipped unless instructed to do so.

A patient has a new prescription for tamsulosin (Flomax) as treatment for benign prostatic hyperplasia. The nurse is checking his current medication list and will contact the prescriber regarding a potential interaction if the patient is also taking which drug? a. Levothyroxine (Synthroid) for hypothyroidism b. Sildenafil (Viagra), an erectile dysfunction medication c. Omeprazole (Prilosec), a proton pump inhibitor d. Low-dose aspirin for stroke prevention

ANS: B Drugs that interact with alpha blockers such as tamsulosin include erectile dysfunction drugs; additive hypotensive effects may occur. The other drugs do not interact with tamsulosin.

An 83-year-old woman has been given a thiazide diuretic to treat mild heart failure. She and her daughter should be told to watch for which problems? a. Constipation and anorexia b. Fatigue, leg cramps, and dehydration c. Daytime sedation and lethargy d. Edema, nausea, and blurred vision

ANS: B Electrolyte imbalance, leg cramps, fatigue, and dehydration are common complications when thiazide diuretics are given to elderly patients. The other options do not describe complications that occur when these drugs are given to the elderly.

A patient wants to take a ginseng dietary supplement. The nurse instructs the patient to look for which potential adverse effect? a. Drowsiness b. Palpitations and anxiety c. Dry mouth d. Constipation

ANS: B Elevated blood pressure, chest pain or palpitations, anxiety, insomnia, headache, nausea, vomiting, and diarrhea are potential adverse effects of ginseng.

A hospitalized patient is experiencing a severe anaphylactic reaction to a dose of intravenous penicillin. Which drug will the nurse expect to use to treat this condition? a. Ephedra b. Epinephrine c. Phenylephrine d. Pseudoephedrine

ANS: B Epinephrine is the drug of choice for the treatment of anaphylaxis. The other drugs listed are incorrect choices.

A patient, diagnosed with lymphoma, has an allergy to one of the proposed chemotherapy drugs. The tumor has not responded to other types of treatment. The nurse expects the oncologist to follow which course of treatment? a. The physician will choose another drug to use. b. The chemotherapy will be given along with supportive measures to treat a possible allergic reaction. c. The patient will receive reduced doses of chemotherapy for a longer period of time. d. The chemotherapy cannot be given because of the patient's allergy.

ANS: B Even if a patient has a known allergic reaction to a given antineoplastic medication, the urgency of treating the patient's cancer may still necessitate administering the medication and then treating any allergic symptoms with supportive medications, such as antihistamines, corticosteroids, and acetaminophen.

During therapy with a beta blocker, the patient notices that she has swollen feet, has gained 3 pounds within 2 days, feels short of breath even when walking around the house, and has been dizzy. The nurse suspects that which of these is occurring? a. The patient is experiencing an allergic reaction. b. The patient may be developing heart failure. c. More time is needed for the patient to see a therapeutic response to the drug. d. The patient is experiencing expected adverse effects of the drug.

ANS: B Even though some beta blockers may be used for the treatment of some types of heart failure, the patient needs to be assessed often for the development of heart failure, a potential adverse effect of the drugs. These symptoms do not indicate expected adverse effects, an allergic reaction, or a therapeutic response.

The nurse is reviewing herbal therapies. Which is a common use of the herb feverfew? a. Muscle aches b. Migraine headaches c. Leg cramps d. Incision pain after surgery

ANS: B Feverfew is commonly used for migraine headaches, menstrual problems, arthritis, and fever. Possible adverse effects include muscle stiffness and muscle and joint pain

A patient is receiving finasteride (Proscar) for treatment of benign prostatic hyperplasia. The nurse will tell him that a possible effect of this medication is: a. alopecia. b. increased hair growth. c. urinary retention. d. increased prostate size.

ANS: B Finasteride is given to reduce prostate size in men with benign prostatic hyperplasia. It has been noted that men taking this medication experience increased hair growth. The other options are incorrect.

A patient who has undergone a lung transplant has contracted cytomegalovirus (CMV) retinitis. The nurse expects which drug to be ordered for this patient? a. Acyclovir (Zovirax) b. Ganciclovir (Cytovene) c. Ribavirin (Virazole) d. Amantadine (Symmetrel)

ANS: B Ganciclovir is indicated for the treatment of cytomegalovirus retinitis. Acyclovir is used for herpes simplex types 1 and 2, herpes zoster, and chickenpox; amantadine is used for influenza type A; and zanamivir is used for influenza types A and B.

A patient is receiving high doses of methotrexate and is experiencing severe bone marrow suppression. The nurse expects which intervention to be ordered with this drug to reduce this problem? a. A transfusion of whole blood b. Leucovorin rescue c. Therapy with filgrastim (Neupogen) d. Administration of allopurinol (Zyloprim)

ANS: B High-dose methotrexate is associated with bone marrow suppression, and it is always given in conjunction with the rescue drug leucovorin, which is an antidote for folic acid antagonists. Basically, leucovorin rescues the healthy cells from methotrexate.

A 57-year-old woman being treated for end-stage breast cancer has been using a transdermal opioid analgesic as part of the management of pain. Lately, she has been experiencing breakthrough pain. The nurse expects this type of pain to be managed by which of these interventions? a. Administering NSAIDs b. Administering an immediate-release opioid c. Changing the opioid route to the rectal route d. Making no changes to the current therapy

ANS: B If a patient is taking long-acting opioid analgesics, breakthrough pain must be treated with an immediate-release dosage form that is given between scheduled doses of the long-acting opioid. The other options are not appropriate actions.

A cancer patient is receiving drug therapy with epoetin alfa (Epogen). The nurse knows that the medication must be stopped if which laboratory result is noted? a. White blood cell count of 550 cells/mm3 b. Hemoglobin level of 12 g/dL c. Potassium level of 4.2 mEq/L d. Glucose level of 78 mg/dL

ANS: B If epoetin is continued when hemoglobin levels are above 11 g/dL, patients may experience serious adverse events, including heart attack, stroke, and death. Guidelines now recommend that the drug be stopped when the hemoglobin level reaches 10 g/dL for cancer patients. For renal patients, the target hemoglobin level is 11 g/dL for patients on dialysis and 10 g/dL for chronic renal patients not on dialysis.

A 73-year-old male patient is in the clinic for a yearly physical and is asking for a prescription for sildenafil (Viagra). He has listed on his health history that he is taking a nitrate for angina. The nurse is aware that which problem may occur if sildenafil is taken with a nitrate? a. Significant increase in pulse rate b. Significant decrease in blood pressure c. Increased risk of bleeding d. Reduced effectiveness of the sildenafil

ANS: B In patients with pre-existing cardiovascular disease, especially those on nitrates, erectile dysfunction drugs such as sildenafil lower blood pressure substantially, potentially leading to more serious adverse events

The nurse is giving an intravenous dose of phenytoin (Dilantin). Which action is correct when administering this drug? a. Give the dose as a fast intravenous (IV) bolus. b. Mix the drug with normal saline, and give it as a slow IV push. c. Mix the drug with dextrose (D5W), and give it as a slow IV push. d. Mix the drug with any available solution as long as the administration rate is correct

ANS: B Intravenous phenytoin is given only with normal saline solution to prevent precipitation formation caused by incompatibilities. The IV push dose must be given slowly (not exceeding 50 mg/min in adults), and the patient must be monitored for bradycardia and decreased blood pressure.

The nurse is administering liquid oral iron supplements. Which intervention is appropriate when administering this medication? a. Have the patient take the liquid iron with milk. b. Instruct the patient to take the medication through a plastic straw. c. Have the patient sip the medication slowly. d. Have the patient drink the medication, undiluted, from the unit-dose cup.

ANS: B Liquid oral forms of iron need to be taken through a plastic straw to avoid discoloration of tooth enamel. Milk may decrease absorption.

A patient in the neurologic intensive care unit is being treated for cerebral edema. Which class of diuretic is used to reduce intracranial pressure? a. Loop diuretics b. Osmotic diuretics c. Thiazide diuretics d. Vasodilators

ANS: B Mannitol, an osmotic diuretic, is commonly used to reduce intracranial pressure and cerebral edema resulting from head trauma.

A patient has been diagnosed with metabolic syndrome and is started on the biguanide metformin (Glucophage). The nurse knows that the purpose of the metformin, in this situation, is which of these? a. To increase the pancreatic secretion of insulin b. To decrease insulin resistance c. To increase blood glucose levels d. To decrease the pancreatic secretion of insulin

ANS: B Metformin decreases glucose production by the liver; decreases intestinal absorption of glucose; and improves insulin receptor sensitivity in the liver, skeletal muscle, and adipose tissue, resulting in decreased insulin resistance.

A patient is experiencing diastolic heart failure. The nurse expects which beta blocker to be ordered for this patient? a. Atenolol (Tenormin) b. Carvedilol (Coreg) c. Acebutolol (Sectral) d. Esmolol (Brevibloc)

ANS: B Not all beta blockers are used for the treatment of heart failure. Carvedilol and metoprolol are currently used in the treatment of heart failure. The other options are not indicated for the treatment of heart failure.

A patient is taking nystatin (Mycostatin) oral lozenges to treat an oral candidiasis infection resulting from inhaled corticosteroid therapy for asthma. Which instruction by the nurse is appropriate? a. "Chew the lozenges until they are completely dissolved." b. "Let the lozenge dissolve slowly and completely in your mouth without chewing it." c. "Rinse your mouth with water before taking the inhaler." d. "Rinse your mouth with mouthwash after taking the inhaler."

ANS: B Nystatin may be given orally in the form of lozenges, or troches, which need to be slowly and completely dissolved in the mouth for optimal effects; tablets are not to be chewed or swallowed whole. The other options are incorrect. Patients taking an inhaled corticosteroid must rinse their mouths with water thoroughly after taking the inhaler.

A patient is receiving heparin therapy as part of the treatment for a pulmonary embolism. The nurse monitors the results of which laboratory test to check the drug's effectiveness? a. Bleeding times b. Activated partial thromboplastin time (aPTT) c. Prothrombin time/international normalized ratio (PT/INR) d. Vitamin K levels

ANS: B Ongoing aPTT values are used to monitor heparin therapy. PT/INR is used to monitor warfarin therapy. The other two options are not used to monitor anticoagulant therapy.

A patient has been treated for lung cancer for 3 years. Over the past few months, the patient has noticed that the opioid analgesic is not helping as much as it had previously and more medication is needed for the same pain relief. The nurse is aware that this patient is experiencing which of these? a. Opioid addiction b. Opioid tolerance c. Opioid toxicity d. Opioid abstinence syndrome

ANS: B Opioid tolerance is a common physiologic result of long-term opioid use. Patients with opioid tolerance require larger doses of the opioid agent to maintain the same level of analgesia. This situation does not describe toxicity (overdose), addiction, or abstinence syndrome (withdrawal).

The wife of a patient who has been diagnosed with depression calls the office and says, "It's been an entire week since he started that new medicine for his depression, and there's no change! What's wrong with him?" What is the nurse's best response? a. "The medication may not be effective for him. He may need to try another type." b. "It may take up to 6 weeks to notice any therapeutic effects. Let's wait a little longer to see how he does." c. "It sounds like the dose is not high enough. I'll check about increasing the dosage." d. "Some patients never recover from depression. He may not respond to this therapy."

ANS: B Patients and family members need to be told that antidepressant drugs commonly require several weeks before full therapeutic effects are noted.

SSRI side effects

GI upset Weight gain or loss Nervousness, agitation or restlessness Dizziness Reduced sexual desire or difficulty reaching orgasm or inability to maintain an erection (erectile dysfunction) Drowsiness Insomnia Headache Dry mouth

Which nursing diagnosis is appropriate for a patient who has started aminoglycoside therapy? a. Constipation b. Risk for injury (renal damage) c. Disturbed body image related to gynecomastia d. Imbalanced nutrition, less than body requirements, related to nausea

ANS: B Patients on aminoglycoside therapy have an increased risk for injury caused by nephrotoxicity. The other options are incorrect.

During a teaching session for a patient on antithyroid drugs, the nurse will discuss which dietary instructions? a. Using iodized salt when cooking b. Avoiding foods containing iodine c. Restricting fluid intake to 2500 mL/day d. Increasing intake of sodium- and potassium-containing foods

ANS: B Patients on antithyroid therapy need to avoid iodine-containing foods. These foods may interfere with the effectiveness of the antithyroid drug.

The nurse is developing a care plan for a patient who is taking an anticholinergic drug. Which nursing diagnosis would be appropriate for this patient? a. Diarrhea b. Urinary retention c. Risk for infection d. Disturbed sleep pattern

ANS: B Patients receiving anticholinergic drugs are at risk for urinary retention and constipation, not diarrhea. The other nursing diagnoses are not applicable to anticholinergic drugs.

The nurse is reviewing the dosage schedule for several different antiepileptic drugs (AEDs). Which antiepileptic drug allows for once-a-day dosing? a. Levetiracetam (Keppra) b. Phenobarbital c. Valproic acid (Depakote) d. Gabapentin (Neurontin)

ANS: B Phenobarbital has the longest half-life of all standard AEDs, including those listed in the other options, so it allows for once-a-day dosing.

The nurse is preparing to administer insulin intravenously. Which statement about the administration of intravenous insulin is true? a. Insulin is never given intravenously. b. Only regular insulin can be administered intravenously. c. Insulin aspart or insulin lispro can be administered intravenously, but there must be a 50% dose reduction. d. Any form of insulin can be administered intravenously at the same dose as that is ordered for subcutaneous administration.

ANS: B Regular insulin is the usual insulin product to be dosed via intravenous bolus, intravenous infusion, or even intramuscularly. These routes, especially the intravenous infusion route, are often used in cases of diabetic ketoacidosis, or coma associated with uncontrolled type 1 diabetes.

A patient is taking chemotherapy with a drug that has a high potential for causing nausea and vomiting. The nurse is preparing to administer an antiemetic drug. Which class of antiemetic drugs is most commonly used to prevent nausea and vomiting for patients receiving chemotherapy? a. Prokinetic drugs, such as metoclopramide (Reglan) b. Serotonin blockers, such as ondansetron (Zofran) c. Anticholinergic drugs, such as scopolamine d. Neuroleptic drugs, such as promethazine (Phenergan)

ANS: B Serotonin blockers used to prevent chemotherapy-induced and postoperative nausea and vomiting.

A patient who has been taking cimetidine (Tagamet) for hyperacidity calls the clinic to say that the medication has not been effective. The nurse reviews his history and notes that which factor may be influencing the effectiveness of this drug? a. He takes the cimetidine with meals. b. He smokes two packs of cigarettes a day. c. He drinks a glass of water with each dose. d. He takes an antacid 3 hours after the cimetidine dose.

ANS: B Smoking may impair the absorption of H2 antagonists.

During a follow-up visit, a patient who has been on estrogen therapy admits that she has continued to smoke cigarettes. The nurse will remind the patient that smoking while on estrogen may lead to increased: a. incidence of nausea. b. risk for thrombosis. c. levels of triglycerides. d. tendency to bleed during menstruation.

ANS: B Smoking should be avoided during estrogen therapy because it adds to the risk for thrombosis formation.

When teaching a patient who is beginning antilipemic therapy about possible drug-food interactions, the nurse will discuss which food? a. Oatmeal b. Grapefruit juice c. Licorice d. Dairy products

ANS: B Taking HMG-CoA reductase inhibitors with grapefruit juice may cause complications. Components in grapefruit juice inactivate CYP3A4 in both the liver and intestines. This enzyme plays a key role in statin metabolism. The presence of grapefruit juice in the body may therefore result in sustained levels of unmetabolized statin drug, which increases the risk for major drug toxicity, possibly leading to rhabdomyolysis

When administering cyclosporine, the nurse notes that allopurinol is also ordered for the patient. What is a potential result of this drug interaction? a. Reduced adverse effects of the cyclosporine b. Increased levels of cyclosporine and toxicity c. Reduced uric acid levels d. Reduced nephrotoxic effects of cyclosporine

ANS: B The allopurinol may cause increased levels of cyclosporine, and toxicity may result

After starting treatment for type 2 diabetes mellitus 6 months earlier, a patient is in the office for a follow-up examination. The nurse will monitor which laboratory test to evaluate the patient's adherence to the antidiabetic therapy over the past few months? a. Hemoglobin levels b. Hemoglobin A1C level c. Fingerstick fasting blood glucose level d. Serum insulin levels

ANS: B The hemoglobin A1C level reflects the patient's adherence to the therapy regimen for several months previously, thus evaluating how well the patient has been doing with diet and drug therapy

A patient who has recently started therapy on a statin drug asks the nurse how long it will take until he sees an effect on his serum cholesterol. Which statement would be the nurse's best response? a. "Blood levels return to normal within a week of beginning therapy." b. "It takes 6 to 8 weeks to see a change in cholesterol levels." c. "It takes at least 6 months to see a change in cholesterol levels." d. "You will need to take this medication for almost a year to see significant results."

ANS: B The maximum extent to which lipid levels are lowered may not occur until 6 to 8 weeks after the start of therapy.

A patient has a digoxin level of 1.1 ng/mL. Which interpretation by the nurse is correct? a. It is below the therapeutic level. b. It is within the therapeutic range. c. It is above the therapeutic level. d. It is at a toxic level.

ANS: B The normal therapeutic drug level of digoxin is between 0.5 and 2 ng/mL.

A patient has had an overdose of an intravenous cholinergic drug. The nurse expects to administer which drug as an antidote? a. Atenolol (Tenormin) b. Bethanechol (Urecholine) c. Dobutamine d. Atropine sulfate

ANS: D Prompt administration of atropine sulfate can reverse a toxic dose of cholinergic drugs. The other drugs listed are not antidotes to cholinergic toxicity.

A patient who is taking the bisphosphonate alendronate (Fosamax) has been instructed to lie flat in bed for 2 days after having ophthalmic surgery. Which intervention is appropriate at this time? a. She will continue to take the alendronate with water. b. She cannot take the alendronate until she can sit up for 30 minutes. c. She can take the medication with breakfast. d. She will stop taking the medication 72 hours before her surgery.

ANS: B The nurse must emphasize that the patient should remain upright in either a standing or sitting position for approximately 30 minutes after taking a bisphosphonate so as to help prevent esophageal erosion or irritation. Because this patient will be required to lie flat in bed for 2 days after the surgery, the prescriber will need to be notified that the patient cannot take the medication during this time

A patient is being treated for ethanol alcohol abuse in a rehabilitation center. The nurse will include which information when teaching him about disulfiram (Antabuse) therapy? a. He should not smoke cigarettes while on this drug. b. He needs to know about the common over-the-counter substances that contain alcohol. c. This drug will cause the same effects as the alcohol did, without the euphoric effects. d. Mouthwashes and cough medicines that contain alcohol are safe because they are used in small amounts.

ANS: B The use of disulfiram (Antabuse) with alcohol-containing over-the-counter products will elicit severe adverse reactions. As little as 7 mL of alcohol may cause symptoms in a sensitive person. Cigarette smoking does not cause problems when taking disulfiram. Disulfiram does not have the same effects as alcohol.

During an assessment, the patient tells the nurse that he eats large amounts of garlic for its cardiovascular benefits. Which drug or drug class, if taken, would have a potential interaction with the garlic? a. Acetaminophen (Tylenol) b. Insulin c. Antilipemic drugs d. Sedatives

ANS: B The use of garlic may interfere with hypoglycemic drugs. The other options are incorrect because acetaminophen, antilipemic drugs, and sedatives do not have interactions with garlic.

A patient wants to take the herb gingko to help his memory. The nurse reviews his current medication list and would be concerned about potential interactions if he is taking a medication from which class of drugs? a. Digitalis b. Anticoagulants c. Sedatives d. Immunosuppressants

ANS: B The use of gingko increases the risk of bleeding with anticoagulants (warfarin, heparin) and antiplatelets (aspirin, clopidogrel).

The nurse is preparing to administer morning medications to a patient who has been newly diagnosed with tuberculosis. The patient asks, "Why do I have to take so many different drugs?" Which response by the nurse is correct? a. "Your prescriber hopes that at least one of these drugs will work to fight the tuberculosis." b. "Taking multiple drugs reduces the chance that the tuberculosis will become drug resistant." c. "Using more than one drug can help to reduce side effects." d. "Using multiple drugs enhances the effect of each drug."

ANS: B The use of multiple medications reduces the possibility that the organism will become drug resistant.

When administering morning medications for a newly admitted patient, the nurse notes that the patient has an allergy to sulfa drugs. There is an order for the sulfonylurea glipizide (Glucotrol). Which action by the nurse is correct? a. Give the drug as ordered 30 minutes before breakfast. b. Hold the drug, and check the order with the prescriber. c. Give a reduced dose of the drug with breakfast. d. Give the drug, and monitor for adverse effects.

ANS: B There is a potential for cross-allergy in patients who are allergic to sulfonamide antibiotics. Although such an allergy is listed as a contraindication by the manufacturer, most clinicians do prescribe sulfonylureas for such patients.

When administering morning medications for a newly admitted patient, the nurse notes that the patient has an allergy to sulfa drugs. There is an order for the sulfonylurea glipizide (Glucotrol). Which action by the nurse is correct? a. Give the drug as ordered 30 minutes before breakfast. b. Hold the drug, and check the order with the prescriber. c. Give a reduced dose of the drug with breakfast. d. Give the drug, and monitor for adverse effects.

ANS: B There is a potential for cross-allergy in patients who are allergic to sulfonamide antibiotics. Although such an allergy is listed as a contraindication by the manufacturer, most clinicians do prescribe sulfonylureas for such patients. The order needs to be clarified.

Before beginning a patient's therapy with selective serotonin reuptake inhibitor (SSRI) antidepressants, the nurse will assess for concurrent use of which medications or medication class? a. Aspirin b. Anticoagulants c. Diuretics d. Nonsteroidal anti-inflammatory drugs

ANS: B Use of selective serotonin reuptake inhibitor (SSRI) antidepressants with warfarin results in an increased anticoagulant effect. SSRI antidepressants do not interact with the other drugs or drug classes listed.

A patient who has been anticoagulated with warfarin (Coumadin) has been admitted for gastrointestinal bleeding. The history and physical examination indicates that the patient may have taken too much warfarin. The nurse anticipates that the patient will receive which antidote? a. Vitamin E b. Vitamin K c. Protamine sulfate d. Potassium chloride

ANS: B Vitamin K is given to reverse the anticoagulation effects of warfarin toxicity. Protamine sulfate is the antidote for heparin overdose.

A patient has been given a prescription for levodopa-carbidopa (Sinemet) for her newly diagnosed Parkinson's disease. She asks the nurse, "Why are there two drugs in this pill?" The nurse's best response reflects which fact? a. Carbidopa allows for larger doses of levodopa to be given. b. Carbidopa prevents the breakdown of levodopa in the periphery. c. There are concerns about drug-food interactions with levodopa therapy that do not exist with the combination therapy. d. Carbidopa is the biologic precursor of dopamine and can penetrate into the central nervous system.

ANS: B When given in combination with levodopa, carbidopa inhibits the breakdown of levodopa in the periphery and thus allows smaller doses of levodopa to be used. Lesser amounts of levodopa result in fewer unwanted adverse effects. Levodopa, not carbidopa, is the biologic precursor of dopamine and can penetrate into the CNS.

A patient tells the nurse that he likes to eat large amounts of garlic "to help lower his cholesterol levels naturally." The nurse reviews his medication history and notes that which drug has a potential interaction with the garlic? a. Acetaminophen (Tylenol) b. Warfarin (Coumadin) c. Digoxin (Lanoxin) d. Phenytoin (Dilantin)

ANS: B When using garlic, it is recommended to avoid any other drugs that may interfere with platelet and clotting function. These drugs include antiplatelet drugs, anticoagulants, nonsteroidal antiinflammatory drugs, and aspirin. The other drugs listed do not have known interactions with garlic.

The nurse is monitoring a patient who has been taking carbamazepine (Tegretol) for 2 months. Which effects would indicate that autoinduction has started to occur? a. The drug levels for carbamazepine are higher than expected. b. The drug levels for carbamazepine are lower than expected. c. The patient is experiencing fewer seizures. d. The patient is experiencing toxic effects from the drug.

ANS: B With carbamazepine, autoinduction occurs and leads to lower than expected drug concentrations. Therefore, the dosage may have to be adjusted with time.

An oral iron supplement is prescribed for a patient. The nurse would question this order if the patient's medical history includes which condition? a. Decreased hemoglobin b. Hemolytic anemia c. Weakness d. Concurrent therapy with erythropoietics

ANS: B Hemolytic anemia is a contraindication to the use of iron supplements. Decreased hemoglobin and weakness are related to iron-deficiency anemia. Iron supplements are given with erythropoietic drugs to aid in the production of red blood cells.

The nurse will monitor for myopathy (muscle pain) when a patient is taking which class of antilipemic drugs? a. Niacin b. HMG-CoA reductase inhibitors c. Fibric acid derivatives d. Bile acid sequestrants

ANS: B Myopathy (muscle pain) is a clinically important adverse effect that may occur with HMG-CoA reductase inhibitors. It may progress to a serious condition known as rhabdomyolysis. Patients receiving statin therapy need to be advised to report any unexplained muscular pain or discomfort to their health care providers immediately. The other drugs and drug classes do not cause muscle pain or myopathy.

Antilipemic drug therapy is prescribed for a patient, and the nurse is providing instructions to the patient about the medication. Which instructions will the nurse include? (Select all that apply.) a. Limit fluid intake to prevent fluid overload. b. Eat extra servings of raw vegetables and fruit. c. Report abnormal or unusual bleeding or yellow discoloration of the skin. d. Report the occurrence of muscle pain immediately. e. Drug interactions are rare with antilipemics. f. Take the drug 1 hour before or 2 hours after meals to maximize absorption.

ANS: B, C, D b. Eat extra servings of raw vegetables and fruit. c. Report abnormal or unusual bleeding or yellow discoloration of the skin. d. Report the occurrence of muscle pain immediately. Instructions need to include preventing constipation by encouraging a diet that is plentiful in raw vegetables, fruit, and bran. Forcing fluids (up to 3000 mL/day unless contraindicated) may also help to prevent constipation. Notify the prescriber if there are any new or troublesome symptoms, abnormal or unusual bleeding, yellow discoloration of the skin, or muscle pain. These drugs are highly protein bound; therefore, they interact with many drugs. Taking these drugs with food may help to reduce gastrointestinal distress.

The nurse is reviewing antiepileptic drug (AED) therapy. Which statements about AED therapy are accurate? (Select all that apply.) a. AED therapy can be stopped when seizures are stopped. b. AED therapy is usually lifelong. c. Consistent dosing is the key to controlling seizures. d. A dose may be skipped if the patient is experiencing adverse effects. e. Do not abruptly discontinue AEDs because doing so may cause rebound seizure activity.

ANS: B, C, E b. AED therapy is usually lifelong. c. Consistent dosing is the key to controlling seizures. e. Do not abruptly discontinue AEDs because doing so may cause rebound seizure activity. Patients need to know that AED therapy is usually lifelong, and compliance (with consistent dosing) is important for effective seizure control. Abruptly stopping AED therapy may cause withdrawal (or rebound) seizure activity.

The nurse is providing patient teaching about antacids. Which statements about antacids are accurate? (Select all that apply.) a. Antacids reduce the production of acid in the stomach. b. Antacids neutralize acid in the stomach. c. Rebound hyperacidity may occur with calcium-based antacids. d. Aluminum-based antacids cause diarrhea. e. Magnesium-based antacids cause diarrhea.

ANS: B, C, E b. Antacids neutralize acid in the stomach. c. Rebound hyperacidity may occur with calcium-based antacids. e. Magnesium-based antacids cause diarrhea. Antacids neutralize acid in the stomach. Magnesium-based antacids cause diarrhea, and aluminum-based antacids cause constipation. Calcium-based antacids often cause rebound hyperacidity.

Vicodin (acetaminophen/hydrocodone) is prescribed for a patient who has had surgery. The nurse informs the patient that which common adverse effects can occur with this medication? (Select all that apply.) a. Diarrhea b. Constipation c. Lightheadedness d. Nervousness e. Urinary retention f. Itching

ANS: B, C, E, F b. Constipation c. Lightheadedness e. Urinary retention f. Itching Constipation (not diarrhea), lightheadedness (not nervousness), urinary retention, and itching are some of the common adverse effects that the patient may experience while taking Vicodin.

The nurse is preparing to administer a barbiturate. Which conditions or disorders would be a contraindication to the use of these drugs? (Select all that apply.) a. Gout b. Pregnancy c. Epilepsy d. Severe chronic obstructive pulmonary disease e. Severe liver disease f. Diabetes mellitus

ANS: B, D, E b. Pregnancy d. Severe chronic obstructive pulmonary disease e. Severe liver disease Contraindications to barbiturates include pregnancy, significant respiratory difficulties, and severe liver disease.

When a patient is experiencing digoxin toxicity, which clinical situation would necessitate the use of digoxin immune Fab (Digifab)? (Select all that apply.) a. The patient reports seeing colorful halos around lights. b. The patient's serum potassium level is above 5 mEq/L. c. The patient is experiencing nausea and anorexia. d. The patient is experiencing severe sinus bradycardia that does not respond to cardiac pacing. e. The patient has received an overdose of greater than 10 mg of digoxin. f. The patient reports fatigue and headaches.

ANS: B, D, E b. The patient's serum potassium level is above 5 mEq/L. d. The patient is experiencing severe sinus bradycardia that does not respond to cardiac pacing. e. The patient has received an overdose of greater than 10 mg of digoxin. Clinical situations that would require the use of digoxin immune Fab in a patient with digoxin toxicity include serum potassium level above 5 mEq/L, severe sinus bradycardia that does not respond to cardiac pacing, or an overdose of more than 10 mg of digoxin. Seeing colorful halos around lights and experiencing nausea, anorexia, fatigue, and headaches are potential adverse effects of digoxin therapy but are not necessarily reasons for digoxin immune Fab treatment.

The nurse is administering an amphotericin B infusion. Which actions by the nurse are appropriate? (Select all that apply.) a. Administering the medication by rapid IV infusion b. Discontinuing the drug immediately if the patient develops tingling and numbness in the extremities c. If adverse effects occur, reducing the IV rate gradually until they subside d. Using an infusion pump for IV therapy e. Monitoring the IV site for signs of phlebitis and infiltration f. Administering premedication for fever and nausea g. Knowing that the intravenous solution for amphotericin B will be cloudy h. Knowing that muscle twitching may indicate hypokalemia

ANS: B, D, E, F b. Discontinuing the drug immediately if the patient develops tingling and numbness in the extremities d. Using an infusion pump for IV therapy e. Monitoring the IV site for signs of phlebitis and infiltration f. Administering premedication for fever and nausea If the patient develops tingling and numbness in the extremities (paresthesias), discontinue the drug immediately. An infusion pump is necessary for the infusion, and the nurse will monitor the IV site for signs of phlebitis and infiltration. Premedication to reduce the adverse effects of fever, malaise, and nausea may be ordered. The IV solution must be clear and without precipitates; and muscle weakness, not twitching, may indicate hypokalemia. The medication must be administered at the rate recommended and stopped, not slowed, if adverse reactions occur.

A patient has been taking disulfiram (Antabuse) as part of his rehabilitation therapy. However, this evening, he attended a party and drank half a beer. As a result, he became ill and his friends took him to the emergency department. The nurse will look for which adverse effects associated with acetaldehyde syndrome? (Select all that apply.) a. Euphoria b. Severe vomiting c. Diarrhea d. Pulsating headache e. Difficulty breathing f. Sweating

ANS: B, D, E, F b. Severe vomiting d. Pulsating headache e. Difficulty breathing f. Sweating Acetaldehyde syndrome results when alcohol is taken while on disulfiram (Antabuse) therapy. Adverse effects include CNS effects (pulsating headache, sweating, marked uneasiness, weakness, vertigo, others); GI effects (nausea, copious vomiting, thirst); and difficulty breathing. Cardiovascular effects also occur; see Table 17-2. Euphoria and diarrhea are not adverse effects associated with acetaldehyde syndrome.

The nurse follows which procedures when giving intravenous (IV) cyclosporine? (Select all that apply.) a. Administering it as a single IV bolus injection to minimize adverse effects b. Using an infusion pump to administer this medication c. Monitoring the patient for potential delayed adverse effects, which may be severe d. Monitoring the patient closely for the first 30 minutes for severe adverse effects e. Checking blood levels periodically during cyclosporine therapy f. Performing frequent oral care during therapy

ANS: B, D, E, F b. Using an infusion pump to administer this medication d. Monitoring the patient closely for the first 30 minutes for severe adverse effects e. Checking blood levels periodically during cyclosporine therapy f. Performing frequent oral care during therapy Cyclosporine is infused intravenously with an infusion pump, not as an IV bolus. Monitor the patient closely for the first 30 minutes for adverse effects, especially for allergic reactions, and monitor blood levels periodically to ensure therapeutic, not toxic, levels of the medication. Perform oral hygiene frequently to prevent dry mouth and subsequent infections.

A patient is on a chemotherapy regimen in an outpatient clinic and is receiving a chemotherapy drug that is known to be highly emetogenic. The nurse will implement which interventions regarding the pharmacologic management of nausea and vomiting? (Select all that apply.) a. Giving antinausea drugs at the beginning of the chemotherapy infusion b. Administering antinausea drugs 30 to 60 minutes before chemotherapy is started c. For best therapeutic effects, medicating for nausea once the symptoms begin d. Observing carefully for the adverse effects of restlessness and anxiety e. Instructing the patient that the antinausea drugs may cause extreme drowsiness f. Instructing the patient to rise slowly from a sitting or lying position because of possible orthostatic hypotension

ANS: B, E, F b. Administering antinausea drugs 30 to 60 minutes before chemotherapy is started e. Instructing the patient that the antinausea drugs may cause extreme drowsiness f. Instructing the patient to rise slowly from a sitting or lying position because of possible orthostatic hypotension Antiemetics should be given before any chemotherapy drug is administered, often 30 to 60 minutes before treatment, but not immediately before chemotherapy is administered. Do not wait until the nausea begins. Most antiemetics cause drowsiness, not restlessness and anxiety. Orthostatic hypotension is a possible adverse effect that may lead to injury.

When admitting a patient with a suspected diagnosis of chronic alcohol use, the nurse will keep in mind that chronic use of alcohol might result in which condition? a. Renal failure b. Cerebrovascular accident c. Korsakoff's psychosis d. Alzheimer's disease

ANS: C A variety of serious neurologic and mental disorders, such as Korsakoff's psychosis and Wernicke's encephalopathy, as well as cirrhosis of the liver, may occur with chronic use of alcohol. Renal failure, cerebrovascular accident, and Alzheimer's disease are not associated directly with chronic use of alcohol.

During a routine checkup, a patient states that she is unable to take the prescribed antihistamine because of one of its most common adverse effects. The nurse suspects that which adverse effect has been bothering this patient? a. Constipation b. Abdominal cramps c. Drowsiness d. Decreased libido

ANS: C Drowsiness is usually the chief complaint of people who take antihistamines.

A patient is started on a diuretic for antihypertensive therapy. The nurse expects that a drug in which class is likely to be used initially? a. Loop diuretics b. Osmotic diuretics c. Thiazide diuretics d. Potassium-sparing diuretics

ANS: C The Eighth Report of the Joint National Committee on Prevention, Detection, Evaluation, and Treatment of High Blood Pressure (JNC-8) guidelines reaffirmed the role of thiazide diuretics as one of the first-line treatment for hypertension. The other drug classes are not considered firstline treatments.

The nurse is assessing a patient who has been admitted to the emergency department for a possible opioid overdose. Which assessment finding is characteristic of an opioid drug overdose? a. Dilated pupils b. Restlessness c. Respiration rate of 6 breaths/min d. Heart rate of 55 beats/min

ANS: C The most serious adverse effect of opioid use is CNS depression, which may lead to respiratory depression. Pinpoint pupils, not dilated pupils, are seen. Restlessness and a heart rate of 55 beats/min are not indications of an opioid overdose.

A patient has been taking temazepam (Restoril) for intermittent insomnia. She calls the nurse to say that when she takes it, she sleeps well, but the next day she feels "so tired." Which explanation by the nurse is correct? a. "Long-term use of this drug results in a sedative effect." b. "If you take the drug every night, this hangover effect will be reduced." c. "These drugs affect the sleep cycle, resulting in daytime sleepiness." d. "These drugs increase the activity of the central nervous system, making you tired the next day."

ANS: C Benzodiazepines suppress REM sleep to a degree (although not as much as barbiturates) and, thus, result in daytime sleepiness (a hangover effect).

The nurse is providing teaching for a patient who is to receive estrogen replacement therapy. Which statement is correct to include in the teaching session? a. "If you miss a dose, double-up on the next dose." b. "There's no need to be concerned about breast lumps or bumps that occur." c. "Be sure to report any weight gain of 5 pounds or more per week." d. "Take the medication on an empty stomach to enhance absorption."

ANS: C Patients taking oral estrogen therapy should report weight gain of 5 pounds or more per week to a physician. The other statements are not true for estrogen replacement therapy.

A patient has received a prescription for a 2-week course of antifungal suppositories for a vaginal yeast infection. She asks the nurse if there is an alternative to this medication, saying, "I don't want to do this for 2 weeks!" Which is a possibility in this situation? a. A single dose of a vaginal antifungal cream. b. A one-time infusion of amphotericin B. c. A single dose of a fluconazole (Diflucan) oral tablet. d. There is no better alternative to the suppositories.

ANS: C A single oral dose of fluconazole may be used to treat vaginal candidiasis. The other options are incorrect.

A patient is to receive acetylcysteine (Mucomyst) as part of the treatment for an acetaminophen (Tylenol) overdose. Which action by the nurse is appropriate when giving this medication? a. Giving the medication undiluted for full effect b. Avoiding the use of a straw when giving this medication c. Disguising the flavor with soda or flavored water d. Preparing to give this medication via a nebulizer

ANS: C Acetylcysteine has the flavor of rotten eggs and so is better tolerated if it is diluted and disguised by mixing with a drink such as cola or flavored water to help increase its palatability. The use of a straw helps to minimize contact with the mucous membranes of the mouth and is recommended. The nebulizer form of this medication is used for certain types of pneumonia, not for acetaminophen overdose.

The nurse is reviewing the food choices of a patient who is taking a monoamine oxidase inhibitor ( MAOI). Which food choice would indicate the need for additional teaching? a. Orange juice b. Fried eggs over-easy c. Salami and Swiss cheese sandwich d. Biscuits and honey

ANS: C Aged cheeses, such a Swiss or cheddar cheese, and Salami contain tyramine. Patients who are taking MAOIs need to avoid tyramine-containing foods because of a severe hypertensive reaction that may occur. Orange juice, eggs, biscuits, and honey do not contain tyramine.

When reviewing the allergy history of a patient, the nurse notes that the patient is allergic to penicillin. Based on this finding, the nurse would question an order for which class of antibiotics? a. Tetracyclines b. Sulfonamides c. Cephalosporins d. Quinolones

ANS: C Allergy to penicillin may also result in hypersensitivity to cephalosporins. The other options are incorrect.

A nurse is giving instructions to a patient who will be receiving oral iron supplements. Which instructions will be included in the teaching plan? a. Take the iron tablets with milk or antacids. b. Crush the pills as needed to help with swallowing. c. Take the iron tablets with meals if gastrointestinal distress occurs. d. If black tarry stools occur, report it to the doctor immediately.

ANS: C Although taking iron tablets with food may decrease absorption, doing so helps to reduce gastrointestinal distress. Antacids and milk may cause decreased iron absorption; iron tablets must be taken whole and not crushed. Black, tarry stools are expected adverse effects of oral iron supplements.

The nurse will monitor for which adverse effect when administering an anticholinergic drug? a. Excessive urination b. Diaphoresis c. Dry mouth d. Pupillary constriction

ANS: C Anticholinergic drugs commonly cause the adverse effects of dry mouth, blurred vision, constipation, and urinary retention. They also cause mydriasis (pupillary dilation).

The nurse notes in the patient's medication orders that the patient will be starting anticoagulant therapy. What is the primary goal of anticoagulant therapy? a. Stabilizing an existing thrombus b. Dissolving an existing thrombus c. Preventing thrombus formation d. Dilating the vessel around a clot

ANS: C Anticoagulants prevent thrombus formation but do not dissolve or stabilize an existing thrombus, nor do they dilate vessels around a clot.

The nurse will teach a patient who is receiving oral iron supplements to watch for which expected adverse effects? a. Palpitations b. Drowsiness and dizziness c. Black, tarry stools d. Orange-red discoloration of the urine

ANS: C Black, tarry stools and other gastrointestinal disturbances may occur with the administration of iron preparations

A patient is suffering from tendonitis of the knee. The nurse is reviewing the patient's medication administration record and recognizes that which adjuvant medication is most appropriate for this type of pain? a. Antidepressant b. Anticonvulsant c. Corticosteroid d. Local anesthesia

ANS: C Corticosteroids have an anti-inflammatory effect, which may help to reduce pain. The other medications do not have anti-inflammatory properties.

The nurse is reviewing medications used for depression. Which of these statements is a reason that selective serotonin reuptake inhibitors (SSRIs) are more widely prescribed today than tricyclic antidepressants? a. SSRIs have fewer sexual side effects. b. Unlike tricyclic antidepressants, SSRIs do not have drug-food interactions. c. Tricyclic antidepressants cause serious cardiac dysrhythmias if an overdose occurs. d. SSRIs cause a therapeutic response faster than tricyclic antidepressants.

ANS: C Death from overdose of tricyclic antidepressants usually results from either seizures or dysrhythmias. SSRIs are associated with significantly fewer and less severe systemic adverse effects, especially anticholinergic and cardiovascular adverse effects.

The nurse is monitoring for therapeutic results of antibiotic therapy in a patient with an infection. Which laboratory value would indicate therapeutic effectiveness of this therapy? a. Increased red blood cell count b. Increased hemoglobin level c. Decreased white blood cell count d. Decreased platelet count

ANS: C Decreased white blood cell counts are an indication of reduction of infection and are a therapeutic effect of antibiotic therapy.

When a patient is on aminoglycoside therapy, the nurse will monitor the patient for which indicators of potential toxicity? a. Fever b. White blood cell count of 8000 cells/mm3 c. Tinnitus and dizziness d. Decreased blood urea nitrogen (BUN) levels

ANS: C Dizziness, tinnitus, hearing loss, or a sense of fullness in the ears could indicate ototoxicity, a potentially serious toxicity in a patient. Nephrotoxicity is indicated by rising blood urea nitrogen and creatinine levels. Fever may be indicative of the patient's infection; a white blood cell count of 7000 cells/mm3 is within the normal range of 5000 to 10,000 cells/mm3 .

A patient calls the clinic to speak to the nurse about taking an herbal product that contains ginkgo (Ginkgo biloba) to "help my memory." He states that he has read much information about the herbal product. Which statement by the patient indicates a need for further education? a. "I know the FDA has not approved this herbal product, but I'd like to try it to see if it helps my memory." b. "I need to watch for possible side effects, such as headaches, or stomach or intestinal upset." c. "I will take aspirin or ibuprofen (Motrin) if I have a headache." d. "Ginkgo may cause increased bleeding, so I'll have to be careful when doing yard work."

ANS: C Drug interactions may occur between the taking of aspirin and nonsteroidal anti-inflammatory drugs and the taking of ginkgo.

A patient who has started drug therapy for tuberculosis wants to know how long he will be on the medications. Which response by the nurse is correct? a. "Drug therapy will last until the symptoms have stopped." b. "Drug therapy will continue until the tuberculosis develops resistance." c. "You should expect to take these drugs for as long as 24 months." d. "You will be on this drug therapy for the rest of your life."

ANS: C Drug therapy commonly lasts for 24 months if consistent drug therapy has been maintained.

When teaching about hypoglycemia, the nurse will make sure that the patient is aware of the early signs of hypoglycemia, including: a. hypothermia and seizures. b. nausea and diarrhea. c. confusion and sweating. d. fruity, acetone odor to the breath.

ANS: C Early symptoms of hypoglycemia include the central nervous system manifestations of confusion, irritability, tremor, and sweating. Hypothermia and seizures are later symptoms of hypoglycemia.

A patient with end-stage renal failure has been admitted to the hospital for severe anemia. She is refusing blood transfusions. The nurse anticipates drug therapy with which drug to stimulate the production of red blood cells? a. Folic acid b. Cyanocobalamin (vitamin B12) c. Epoetin alfa (Epogen) d. Filgrastim (Neupogen)

ANS: C Epoetin alfa is a colony-stimulating factor that is responsible for erythropoiesis, or formation of red blood cells

When monitoring patients on antitubercular drug therapy, the nurse knows that which drug may cause a decrease in visual acuity? a. Rifampin (Rifadin) b. Isoniazid (INH) c. Ethambutol (Myambutol) d. Streptomycin

ANS: C Ethambutol may cause a decrease in visual acuity or even blindness resulting from retrobulbar neuritis. The other options are incorrect.

A 75-year-old woman comes into the clinic with complaints of muscle twitching, nausea, and headache. She tells the nurse that she has been taking sodium bicarbonate five or six times a day for the past 3 weeks. The nurse will assess for which potential problem that may occur with overuse of sodium bicarbonate? a. Constipation b. Metabolic acidosis c. Metabolic alkalosis d. Excessive gastric mucus

ANS: C Excessive use of sodium bicarbonate may lead to systemic alkalosis.

A patient has been taking haloperidol (Haldol) for 3 months for a psychotic disorder, and the nurse is concerned about the development of extrapyramidal symptoms. The nurse will monitor the patient closely for which effects? a. Increased paranoia b. Drowsiness and dizziness c. Tremors and muscle twitching d. Dry mouth and constipation

ANS: C Extrapyramidal symptoms are manifested by tremors and muscle twitching, and the incidence of such symptoms is high during haloperidol therapy

A patient is in the intensive care unit and receiving an infusion of milrinone (Primacor) for severe heart failure. The prescriber has written an order for an intravenous dose of furosemide (Lasix). How will the nurse give this drug? a. Infuse the drug into the same intravenous line as the milrinone. b. Stop the milrinone, flush the line, and then administer the furosemide. c. Administer the furosemide in a separate intravenous line. d. Notify the prescriber that the furosemide cannot be given at this time.

ANS: C Furosemide must not be injected into an intravenous line with milrinone because it will precipitate immediately. The infusion must not be stopped because of the patient's condition. A separate line will be needed.

A patient will be discharged with a 1-week supply of an opioid analgesic for pain management after abdominal surgery. The nurse will include which information in the teaching plan? a. How to prevent dehydration due to diarrhea b. The importance of taking the drug only when the pain becomes severe c. How to prevent constipation d. The importance of taking the drug on an empty stomach

ANS: C Gastrointestinal (GI) adverse effects, such as nausea, vomiting, and constipation, are the most common adverse effects associated with opioid analgesics. Physical dependence usually occurs in patients undergoing long-term treatment. Diarrhea is not an effect of opioid analgesics. Taking the dose with food may help minimize GI upset.

A 38-year-old man has come into the urgent care center with severe hip pain after falling from a ladder at work. He says he has taken several pain pills over the past few hours but cannot remember how many he has taken. He hands the nurse an empty bottle of acetaminophen (Tylenol). The nurse is aware that the most serious toxic effect of acute acetaminophen overdose is which condition? a. Tachycardia b. Central nervous system depression c. Hepatic necrosis d. Nephropathy

ANS: C Hepatic necrosis is the most serious acute toxic effect of an acute overdose of acetaminophen. The other options are incorrect.

A patient is receiving his third intravenous dose of a penicillin drug. He calls the nurse to report that he is feeling "anxious" and is having trouble breathing. What will the nurse do first? a. Notify the prescriber. b. Take the patient's vital signs. c. Stop the antibiotic infusion. d. Check for allergies.

ANS: C Hypersensitivity reactions are characterized by wheezing; shortness of breath; swelling of the face, tongue, or hands; itching; or rash. The nurse should immediately stop the antibiotic infusion, have someone notify the prescriber, and stay with the patient to monitor the patient's vital signs and condition. Checking for allergies should have been done before the infusion

During therapy with the hematopoietic drug epoetin alfa (Epogen), the nurse instructs the patient about adverse effects that may occur, such as: a. anxiety. b. drowsiness. c. hypertension. d. constipation.

ANS: C Hypertension is an adverse effect of hematopoietic drugs, along with headache, fever, pruritus, rash, nausea, vomiting, arthralgia, cough, and injection site reaction

The nurse is reviewing instructions for a patient with type 2 diabetes who also takes insulin injections as part of the therapy. The nurse asks the patient, "What should you do if your fasting blood glucose is 47 mg/dL?" Which response by the patient reflects a correct understanding of insulin therapy? a. "I will call my doctor right away." b. "I will give myself the regular insulin." c. "I will take an oral form of glucose." d. "I will rest until the symptoms pass."

ANS: C Hypoglycemia can be reversed if the patient eats glucose tablets or gel, corn syrup, or honey, or drinks fruit juice or a nondiet soft drink or other quick sources of glucose, which must always be kept at hand. She should not wait for instructions from her physician, nor delay taking the glucose by resting. The regular insulin would only lower her blood glucose levels more.

A patient has a 9-year history of a seizure disorder that has been managed well with oral phenytoin (Dilantin) therapy. He is to be NPO (consume nothing by mouth) for surgery in the morning. What will the nurse do about his morning dose of phenytoin? a. Give the same dose intravenously. b. Give the morning dose with a small sip of water. c. Contact the prescriber for another dosage form of the medication. d. Notify the operating room that the medication has been withheld.

ANS: C If there are any questions about the medication order or the medication prescribed, contact the prescriber immediately for clarification and for an order of the appropriate dose form of the medication. Do not change the route without the prescriber's order. There is an increased risk of seizure activity if one or more doses of the AED are missed.

A patient is to receive iron dextran injections. Which technique is appropriate when the nurse is administering this medication? a. Intravenous administration mixed with 5% dextrose b. Intramuscular injection in the upper arm c. Intramuscular injection using the Z-track method d. Subcutaneous injection into the abdomen

ANS: C Intramuscular iron is given using the Z-track method deep into a large muscle mass. If given intravenously, it is given with normal saline, not 5% dextrose.

A patient in the emergency department was showing signs of hypoglycemia and had a fingerstick glucose level of 34 mg/dL. The patient has just become unconscious. What is the nurse's next action? a. Have the patient eat glucose tablets. b. Have the patient consume fruit juice, a nondiet soft drink, or crackers. c. Administer intravenous glucose (50% dextrose). d. Call the lab to order a fasting blood glucose level.

ANS: C Intravenous glucose raises blood glucose levels when the patient is unconscious and unable to take oral forms of glucose.

A patient with a long-term intravenous catheter is going home. The nurse knows that if he is allergic to seafood, which antiseptic agent is contraindicated? a. Chlorhexidine gluconate (Hibiclens) b. Hydrogen peroxide c. Povidone-iodine (Betadine) d. Isopropyl alcohol

ANS: C Iodine compounds are contraindicated in patients with allergies to seafood.

When reviewing the medication profile of a patient with a new order for desmopressin (DDAVP), the nurse notes that a drug interaction will occur if which drug is taken with desmopressin? a. Aspirin b. Digoxin c. Lithium d. Penicillin

ANS: C Lithium may cause a decreased therapeutic effect of desmopressin. The other options are incorrect.

At 0900, the nurse is about to give morning medications, and the patient has asked for a dose of antacid for severe heartburn. Which schedule for the antacid and medications is correct? a. Give both the antacid and medications at 0900. b. Give the antacid at 0900, and then the medications at 0930. c. Give the medications at 0900, and then the antacid at 1000. d. Give the medications at 0900, and then the antacid at 0915.

ANS: C Medications are not to be taken, unless prescribed, within 1 to 2 hours of taking an antacid because of the impact on the absorption of many medications in the stomach.

A patient who has been on methotrexate therapy is experiencing mild pain. The patient is asking for aspirin for the pain. The nurse recognizes that which of these is true in this situation? a. The aspirin will aggravate diarrhea. b. The aspirin will masks signs of infection. c. Aspirin can lead to methotrexate toxicity. d. The aspirin will cause no problems for the patient on methotrexate. ANS: C Methotrexate interacts with weak organic acids, such as aspirin, and can lead to toxicity by displacing the methotrexate from protein-binding sites.

ANS: C Methotrexate interacts with weak organic acids, such as aspirin, and can lead to toxicity by displacing the methotrexate from protein-binding sites.

A 78-year-old patient is in the recovery room after having a lengthy surgery on his hip. As he is gradually awakening, he requests pain medication. Within 10 minutes after receiving a dose of morphine sulfate, he is very lethargic and his respirations are shallow, with a rate of 7 per minute. The nurse prepares for which priority action at this time? a. Assessment of the patient's pain level b. Immediate intubation and artificial ventilation c. Administration of naloxone (Narcan) d. Close observation of signs of opioid tolerance

ANS: C Naloxone, an opioid-reversal agent, is used to reverse the effects of acute opioid overdose and is the drug of choice for reversal of opioid-induced respiratory depression. This situation is describing an opioid overdose, not opioid tolerance. Intubation and artificial ventilation are not appropriate because the patient is still breathing at 7 breaths/min. It would be inappropriate to assess the patient's level of pain.

The nurse is administering one of the lipid formulations of amphotericin B. When giving this drug, which concept is important to remember? a. The lipid formulations may be given in oral form. b. The doses are much lower than the doses of the older drugs. c. The lipid formulations are associated with fewer adverse effects than the older drugs. d. There is no difference in cost between the newer and older forms.

ANS: C Newer lipid formulations of amphotericin B have been developed in an attempt to decrease the incidence of its adverse effects and increase its efficacy. However, the lipid formulations are more costly.

A patient who has received chemotherapy has a critically low platelet count. The nurse expects which drug or drug class to be used to stimulate platelet cell production? a. Filgrastim (Neupogen) b. Interferons c. Oprelvekin (Neumega) d. Epoetin alfa (Epogen)

ANS: C Oprelvekin (Neumega) stimulates bone marrow cells, specifically megakaryocytes, which eventually form platelets

The nurse checks the patient's laboratory work prior to administering a dose of vancomycin (Vancocin) and finds that the trough vancomycin level is 24 mcg/mL. What will the nurse do next? a. Administer the vancomycin as ordered. b. Hold the drug, and administer 4 hours later. c. Hold the drug, and notify the prescriber. d. Repeat the test to verify results.

ANS: C Optimal blood levels of vancomycin are a trough level of 10 to 20 mcg/mL. Measurement of peak levels is no longer routinely recommended, and only trough levels are commonly monitored. Blood samples for measurement of trough levels are drawn immediately before administration of the next dose. Because of the increase in resistant organisms, many clinicians use a trough level of 15 to 20 mcg/mL as their goal. These trough levels mean that even just before the next dose is due, when drug levels should be low, the drug levels are actually too high.

When evaluating a patient who is taking orlistat (Xenical), which is an intended therapeutic effect? a. Increased wakefulness b. Increased appetite c. Decreased weight d. Decreased hyperactivity

ANS: C Orlistat (Xenical) is a nonstimulant drug that is used as part of a weight loss program.

A patient is receiving oxytocin (Pitocin) to induce labor. During administration of this medication, the nurse will also implement which action? a. Giving magnesium sulfate along with the oxytocin b. Administering the medication in an intravenous (IV) bolus c. Administering the medication with an IV infusion pump d. Monitoring fetal heart rate and maternal vital signs every 6 hours

ANS: C Oxytocin is infused via an infusion pump, not via an IV bolus. Magnesium sulfate is not administered with oxytocin. Fetal heart rate and maternal vital signs should be monitored continuously.

A patient has received an overdose of intravenous heparin, and is showing signs of excessive bleeding. Which substance is the antidote for heparin overdose? a. Vitamin E b. Vitamin K c. Protamine sulfate d. Potassium chloride

ANS: C Protamine sulfate is a specific heparin antidote and forms a complex with heparin, completely reversing its anticoagulant properties. Vitamin K is the antidote for warfarin (Coumadin) overdose.

A patient who has been taking isoniazid (INH) has a new prescription for pyridoxine. She is wondering why she needs this medication. The nurse explains that pyridoxine is often given concurrently with the isoniazid to prevent which condition? a. Hair loss b. Renal failure c. Peripheral neuropathy d. Heart failure

ANS: C Pyridoxine (vitamin B6) may be beneficial for isoniazid-induced peripheral neuropathy. The other options are incorrect.

The nurse is reviewing the medication administration record of a patient who is taking isoniazid (INH). Which drug or drug class has a significant drug interaction with isoniazid? a. Pyridoxine (vitamin B6) b. Penicillins c. Phenytoin (Dilantin) d. Benzodiazepines

ANS: C Taking INH with phenytoin will cause decreased metabolism of the phenytoin, leading to increased drug effects. Pyridoxine is often given with isoniazid to prevent peripheral neuropathy. The other options are incorrect.

When monitoring a patient's response to oral antidiabetic drugs, the nurse knows that which laboratory result would indicate a therapeutic response? a. Random blood glucose level 180 mg/dL b. Blood glucose level of 50 mg/dL after meals c. Fasting blood glucose level between 92 mg/dL d. Evening blood glucose level below 80 mg/dL

ANS: C The American Diabetes Association recommends a fasting blood glucose level of between 80 and 130 mg/dL for diabetic patients.

When reviewing the mechanisms of action of diuretics, the nurse knows that which statement is true about loop diuretics? a. They work by inhibiting aldosterone. b. They are very potent, having a diuretic effect that lasts at least 6 hours. c. They have a rapid onset of action and cause rapid diuresis. d. They are not effective when the creatinine clearance decreases below 25 mL/min.

ANS: C The loop diuretics have a rapid onset of action; therefore, they are useful when rapid onset is desired. Their effect lasts for about 2 hours, and a distinct advantage they have over thiazide diuretics is that their diuretic action continues even when creatinine clearance decreases below 25 mL/min.

When considering the various types of contraceptive drugs, the nurse is aware that which type most closely duplicates the normal hormonal levels of the female menstrual cycle? a. Monophasic b. Biphasic c. Triphasic d. Short acting

ANS: C The triphasic drugs most closely duplicate the normal hormonal levels of the female menstrual cycle.

When treating patients with medications for Parkinson's disease, the nurse knows that the wearing-off phenomenon occurs for which reason? a. There are rapid swings in the patient's response to levodopa. b. The patient cannot tolerate the medications at times. c. The medications begin to lose effectiveness against Parkinson's disease. d. The patient's liver is no longer able to metabolize the drug.

ANS: C The wearing-off phenomenon occurs when antiparkinson medications begin to lose their effectiveness, despite maximal dosing, as the disease progresses.

A patient is brought to the emergency department for treatment of a suspected overdose. The patient was found with an empty prescription bottle of a barbiturate by his bedside. He is lethargic and barely breathing. The nurse would expect which immediate intervention? a. Starting an intravenous infusion of diluted bicarbonate solution b. Administering medications to increase blood pressure c. Implementing measures to maintain the airway and support respirations d. Administrating naloxone (Narcan) as an antagonist

ANS: C There are no antagonists/antidotes for barbiturates. Treatment supports respirations and maintains the airway. The other interventions are not appropriate.

A woman who is in the first trimester of pregnancy has been experiencing severe morning sickness. She asks, "I've heard that ginger tablets may be a natural way to ease the nausea and vomiting. Is it okay to try them?" What is the nurse's best response? a. "They are a safe and natural remedy for nausea when you are pregnant." b. "Go ahead and try them, but stop taking them once the nausea is relieved." c. "Some health care providers do not recommend ginger during pregnancy. Let's check with your provider." d. "You will need to wait until after the first trimester to try them."

ANS: C There is some anecdotal evidence that ginger may have abortifacient properties, and for this reason some clinicians do not recommend its use during pregnancy

The teaching for a patient who is taking tamsulosin (Flomax) to reduce urinary obstruction due to benign prostatic hyperplasia will include which of these? a. Fluids need to be restricted while on this medication. b. Take the medication with breakfast to promote the maximum effects of the drug. c. Get up slowly from a sitting or lying position. d. Blood pressure must be monitored because the medication may cause hypertension.

ANS: C This medication is used to relieve impaired urinary flow in men with benign prostatic hyperplasia, but it also can cause orthostatic hypotension when changing positions from sitting or lying positions. Because of these effects, the blood pressure may become dramatically lowered, and lightheadedness may occur, increasing the risk of falling.

A patient who is diagnosed with shingles is taking topical acyclovir, and the nurse is providing instructions about adverse effects. The nurse will discuss which adverse effects of topical acyclovir therapy? a. Insomnia and nervousness b. Temporary swelling and rash c. Burning when applied d. This medication has no adverse effects.

ANS: C Transient burning may occur with topical application of acyclovir.

When a patient is receiving diuretic therapy, which of these assessment measures would best reflect the patient's fluid volume status? a. Blood pressure and pulse b. Serum potassium and sodium levels c. Intake, output, and daily weight d. Measurements of abdominal girth and calf circumference

ANS: C Urinary intake and output and daily weights are the best reflections of a patient's fluid volume status.

The patient wants to take the herb valerian to help him rest at night. The nurse would be concerned about potential interactions if he is taking a medication from which class of drugs? a. Digitalis b. Anticoagulants c. Sedatives d. Immunosuppressants

ANS: C Valerian may cause increased central nervous system depression if used with sedatives. Digitalis, anticoagulants, and immunosuppressants do not have interactions with valerian

A patient who has been hospitalized for 2 weeks has developed a pressure ulcer that contains multidrug-resistant Staphylococcus aureus (MRSA). Which drug would the nurse expect to be chosen for therapy? a. Metronidazole (Flagyl) b. Ciprofloxacin (Cipro) c. Vancomycin (Vancocin) d. Tobramycin (Nebcin)

ANS: C Vancomycin is the drug of choice for the treatment of MRSA. The other drugs are not used for MRSA

A patient who has type 2 diabetes is scheduled for an oral endoscopy and has been NPO (nothing by mouth) since midnight. What is the best action by the nurse regarding the administration of her oral antidiabetic drugs? a. Administer half the original dose. b. Withhold all medications as ordered. c. Contact the prescriber for further orders. d. Give the medication with a sip of water.

ANS: C When the diabetic patient is NPO, the prescriber needs to be contacted for further orders regarding the administration of the oral antidiabetic drugs.

The nurse is counseling a woman who will be starting rifampin (Rifadin) as part of antitubercular therapy. The patient is currently taking oral contraceptives. Which statement is true regarding rifampin therapy for this patient? a. Women have a high risk for thrombophlebitis while on this drug. b. A higher dose of rifampin will be necessary because of the contraceptive. c. Oral contraceptives are less effective while the patient is taking rifampin. d. The incidence of adverse effects is greater if the two drugs are taken together. x

ANS: C Women taking oral contraceptives and rifampin need to be counseled about other forms of birth control because of the impaired effectiveness of the oral contraceptives during concurrent use of rifampin.

Which statements are true regarding the selective serotonin reuptake inhibitors (SSRIs)? (Select all that apply.) a. Avoid foods and beverages that contain tyramine. b. Monitor the patient for extrapyramidal symptoms. c. Therapeutic effects may not be seen for about 4 to 6 weeks after the medication is started. d. If the patient has been on an MAOI, a 2- to 5-week or longer time span is required before beginning an SSRI medication. e. These drugs have anticholinergic effects, including constipation, urinary retention, dry mouth, and blurred vision. f. Cogentin is often also prescribed to reduce the adverse effects that may occur.

ANS: C, D c. Therapeutic effects may not be seen for about 4 to 6 weeks after the medication is started. d. If the patient has been on an MAOI, a 2- to 5-week or longer time span is required before beginning an SSRI medication. During SSRI medication, therapeutic effects may not be seen for 4 to 6 weeks. To prevent the potentially fatal pharmacodynamic interactions that can occur between the SSRIs and the MAOIs, a 2- to 5-week washout period is recommended between uses of these two classes of medications. The other options apply to other classes of psychotherapeutic drugs, not SSRIs.

When teaching a patient who will be receiving antihistamines, the nurse will include which instructions? (Select all that apply.) a. "Antihistamines are generally safe to take with over-the-counter medications." b. "Take the medication on an empty stomach to maximize absorption of the drug." c. "Take the medication with food to minimize gastrointestinal distress." d. "Drink extra fluids if possible." e. "Antihistamines may cause restlessness and disturbed sleep." f. "Avoid activities that require alertness until you know how adverse effects are tolerated."

ANS: C, D, F c. "Take the medication with food to minimize gastrointestinal distress." d. "Drink extra fluids if possible." f. "Avoid activities that require alertness until you know how adverse effects are tolerated." Antihistamines should be taken with food, even though this slightly reduces the absorption of the drug, so as to minimize the gastrointestinal upset that can occur. Over-the-counter medications must not be taken with an antihistamine unless approved by the physician because of the serious drug interactions that may occur. Drinking extra fluids will help to ease the removal of secretions, and activities that require alertness, such as driving, must not be engaged in until the patient knows how he or she responds to the sedating effects of antihistamines.

A 75-year-old woman has been given a nonsteroidal anti-inflammatory drug (an NSAID for the treatment of rheumatoid arthritis. The nurse is reviewing the patient's medication history and notes that which types of medications could have an interaction with the NSAID? (Select all that apply.) a. Antibiotics b. Decongestants c. Anticoagulants d. Beta blockers e. Diuretics f. Corticosteroids

ANS: C, E, F c. Anticoagulants e. Diuretics f. Corticosteroids Anticoagulants taken with NSAIDs may cause increased bleeding tendencies because of platelet inhibition and hypoprothrombinemia. NSAIDs taken with diuretics may cause reduced hypotensive and diuretic effects. NSAIDs taken with corticosteroids may cause increased ulcerogenic effects.

The nurse will assess the patient for which potential contraindication to antitubercular therapy? a. Glaucoma b. Anemia c. Heart failure d. Hepatic impairment

ANS: D Results of liver function studies (e.g., bilirubin level, liver enzyme levels) need to be assessed because isoniazid and rifampin may cause hepatic impairment; severe liver dysfunction is a contraindication to these drugs. In addition, the patient's history of alcohol use needs to be assessed.

The nurse is preparing an infusion of amphotericin B for a patient who has a severe fungal infection. Which intervention is appropriate regarding the potential adverse effects of amphotericin B? a. Discontinuing the infusion immediately if fever, chills, or nausea occur b. Gradually increasing the infusion rate until the expected adverse effects occur c. If fever, chills, or nausea occur during the infusion, administering medications to treat the symptoms d. Before beginning the infusion, administering an antipyretic and an antiemetic drug

ANS: D Almost all patients given the drug intravenously experience fever, chills, hypotension, tachycardia, malaise, muscle and joint pain, anorexia, nausea and vomiting, and headache. For this reason, pretreatment with an antipyretic (acetaminophen), antihistamines, and antiemetics may be conducted to decrease the severity of the infusion-related reaction.

When teaching a patient about taking a newly prescribed antiepileptic drug (AED) at home, the nurse will include which instruction? a. "Driving is allowed after 2 weeks of therapy." b. "If seizures recur, take a double dose of the medication." c. "Antacids can be taken with the AED to reduce gastrointestinal adverse effects." d. "Regular, consistent dosing is important for successful treatment."

ANS: D Consistent dosing, taken regularly at the same time of day, at the recommended dose, and with meals to reduce the common gastrointestinal adverse effects, is the key to successful management of seizures when taking AEDs. Noncompliance is the factor most likely to lead to treatment failure.

The order reads, "Give levothyroxine (Synthroid), 200 mg, PO once every morning." Which action by the nurse is correct? a. Give the medication as ordered. b. Change the dose to 200 mcg because that is what the prescriber meant. c. Hold the drug until the prescriber returns to see the patient. d. Question the order because the dose is higher than 200 mcg.

ANS: D Levothyroxine is dosed in micrograms. A common medication error is to write the intended dose in milligrams instead of micrograms. If not caught, this error would result in a thousandfold overdose. Doses higher than 200 mcg need to be questioned in case this error has occurred.

Cyclosporine is prescribed for a patient who had an organ transplant. The nurse will monitor the patient for which common adverse effect? a. Nausea and vomiting b. Fever and tremors c. Agitation d. Hypertension

ANS: D Moderate hypertension may occur in as much as 50% of patients taking cyclosporine. The other options are potential adverse effects of other immunosuppressant drugs.

A patient has been taking levothyroxine (Synthroid) for more than one decade for primary hypothyroidism. Today she calls because she has a cousin who can get her the same medication in a generic form from a pharmaceutical supply company. Which is the nurse's best advice? a. "This would be a great way to save money." b. "There's no difference in brands of this medication." c. "This should never be done; once you start with a certain brand, you must stay with it." d. "It's better not to switch brands unless we check with your doctor."

ANS: D Switching brands of levothyroxine during treatment can destabilize the course of treatment. Thyroid function test results need to be monitored more carefully when switching products.

A patient tells the nurse that he likes to drink kava herbal tea to help him relax. Which statement by the patient indicates that additional teaching about this herbal product is needed? a. "I will not drink wine with the kava tea." b. "If I notice my skin turning yellow, I will stop taking the tea." c. "I will not take sleeping pills if I have this tea in the evening." d. "I will be able to drive my car after drinking this tea."

ANS: D Patients should not drive after drinking this tea because it may cause sedation. Kava tea may cause skin discoloration (with long-term use). In addition, it must not be taken with alcohol, barbiturates, and psychoactive drugs.

A patient has been taking antitubercular therapy for 3 months. The nurse will assess for what findings that indicate a therapeutic response to the drug therapy? a. The chronic cough is gone. b. There are two consecutive negative purified protein derivative (PPD) results over 2 months. c. There is increased tolerance to the medication therapy, and there are fewer reports of adverse effects. d. There is a decrease in symptoms of tuberculosis along with improved chest x-rays and sputum cultures.

ANS: D A therapeutic response to antitubercular therapy is manifested by a decrease in the symptoms of tuberculosis, such as cough and fever, and by weight gain. The results of laboratory studies (culture and sensitivity tests) and the chest radiographic findings will be used to confirm the clinical findings of resolution of the infection.

The nurse working in a preoperative admitting unit administers an anticholinergic medication to a patient before surgery. What is the purpose of this drug in the preoperative setting? a. Control the heart rate b. Relax the patient c. Reduce urinary frequency d. Reduce oral and gastrointestinal secretions

ANS: D Anticholinergic drugs are given preoperatively to control oral and gastrointestinal secretions during surgery.

The nurse is reviewing the use of anticholinergic drugs. Anticholinergic drugs block the effects of which nervous system? a. Central nervous system b. Somatic nervous system c. Sympathetic nervous system d. Parasympathetic nervous system

ANS: D Anticholinergic drugs block or inhibit the actions of acetylcholine in the parasympathetic nervous system.

While a patient is receiving antilipemic therapy, the nurse knows to monitor the patient closely for the development of which problem? a. Neutropenia b. Pulmonary problems c. Vitamin C deficiency d. Liver dysfunction

ANS: D Antilipemic drugs may adversely affect liver function; therefore, liver function studies need to be closely monitored. The other options do not reflect problems that may occur with antilipemic drugs.

A 58-year-old man has had a myocardial infarction (MI), has begun rehabilitation, and is ready for discharge. He is given a prescription for metoprolol (Lopressor) and becomes upset after reading the patient education pamphlet. "I don't have high blood pressure—why did my doctor give me this medicine?" Which explanation by the nurse is correct? a. "This medication will prevent blood clots that may lead to another heart attack." b. "Beta blockers will improve blood flow to the kidneys." c. "This drug is prescribed to prevent the high blood pressure that often occurs after a heart attack." d. "Studies have shown that this medication has greatly increased survival rates in patients who have had a heart attack."

ANS: D Beta blockers are frequently given to patients after they have suffered an MI because of their cardioprotective properties.

The nurse is reviewing discharge teaching for a patient who will be taking digoxin (Lanoxin) therapy. The nurse will teach the patient to avoid which foods when taking the digoxin? a. Leafy green vegetables b. Dairy products c. Grapefruit juice d. Bran muffins

ANS: D Bran, in large amounts, may decrease the absorption of oral digitalis drugs. The other foods do not affect digoxin levels.

A 79-year-old patient is taking a diuretic for treatment of hypertension. This patient is very independent and wants to continue to live at home. The nurse will know that which teaching point is important for this patient? a. He should take the diuretic with his evening meal. b. He should skip the diuretic dose if he plans to leave the house. c. If he feels dizzy while on this medication, he needs to stop taking it and take potassium supplements instead. d. He needs to take extra precautions when standing up because of possible orthostatic hypotension and resulting injury from falls.

ANS: D Caution must be exercised in the administration of diuretics to the older adults because they are more sensitive to the therapeutic effects of these drugs and are more sensitive to the adverse effects of diuretics, such as dehydration, electrolyte loss, dizziness, and syncope. Taking the diuretic with the evening meal may disrupt sleep because of nocturia. Doses should never be skipped or stopped without checking with the prescriber

A 6-year-old boy has been started on an extended-release form of methylphenidate hydrochloride (Ritalin) for the treatment of attention deficit hyperactivity disorder (ADHD). During a followup visit, his mother tells the nurse that she has been giving the medication at bedtime so that it will be "in his system" when he goes to school the next morning. What is the nurse's appropriate evaluation of the mother's actions? a. She is giving him the medication dosage appropriately. b. The medication should not be taken until he is at school. c. The medication should be taken with meals for optimal absorption. d. The medication should be given 4 to 6 hours before bedtime to diminish insomnia.

ANS: D Central nervous system stimulants should be taken 4 to 6 hours before bedtime to decrease insomnia. Generally speaking, once-a-day dosing is used with extended-release or long-acting preparations. These formulations eliminate the need to take this medication at school.

Carbidopa-levodopa (Sinemet) is prescribed for a patient with Parkinson's disease. The nurse informs the patient that which common adverse effects can occur with this medication? a. Drowsiness, headache, weight loss b. Dizziness, insomnia, nausea c. Peripheral edema, fatigue, syncope d. Heart palpitations, hypotension, urinary retention

ANS: D Common adverse reactions associated with carbidopa-levodopa include palpitations, hypotension, urinary retention, dyskinesia, and depression. The other effects may occur with other antiparkinson drugs.

The nurse is reviewing the medication orders for a patient who will be receiving gentamicin therapy. Which other medication or medication class, if ordered, would be a potential interaction concern? a. Calcium channel blockers b. Phenytoin c. Proton pump inhibitors d. Loop diuretics

ANS: D Concurrent use of aminoglycosides, such as gentamicin, with loop diuretics increases the risk for ototoxicity. The other drugs and drug classes do not cause interactions.

A patient has a prescription for oxybutynin (Ditropan), an anticholinergic drug. When reviewing the patient's medical history, which condition, if present, would be considered a contraindication to therapy with this drug? a. Diarrhea b. Hypertension c. Neurogenic bladder d. Uncontrolled angle-closure glaucoma

ANS: D Contraindications include drug allergy, urinary or gastric retention, and uncontrolled angleclosure glaucoma. Neurogenic bladder is an indication for oxybutynin. The other options are incorrect.

A patient, newly diagnosed with hypothyroidism, receives a prescription for a thyroid hormone replacement drug. The nurse assesses for which potential contraindication to this drug? a. Infection b. Diabetes mellitus c. Liver disease d. Recent myocardial infarction

ANS: D Contraindications to thyroid preparations include known drug allergy to a given drug product, recent myocardial infarction, adrenal insufficiency, and hyperthyroidism.

A patient has been receiving epoetin alfa (Epogen) for severe iron-deficiency anemia. Today, the provider changed the order to darbepoetin (Aranesp). The patient questions the nurse, "What is the difference in these drugs?" Which response by the nurse is correct? a. "There is no difference in these two drugs." b. "Aranesp works faster than Epogen to raise your red blood cell count." c. "Aranesp is given by mouth, so you will not need to have injections." d. "Aranesp is a longer-acting form, so you will receive fewer injections."

ANS: D Darbepoetin (Aranesp) is longer-acting than epoetin alfa (Epogen); therefore, fewer injections are required.

A patient about to receive a morning dose of digoxin has an apical pulse of 53 beats/min. What will the nurse do next? a. Administer the dose. b. Administer the dose, and notify the prescriber. c. Check the radial pulse for 1 full minute. d. Withhold the dose, and notify the prescriber.

ANS: D Digoxin doses are held and the prescriber notified if the apical pulse is 60 beats/min or lower or is higher than 100 beats/min.

A patient has been taking digoxin at home but took an accidental overdose and has developed toxicity. The patient has been admitted to the telemetry unit, where the physician has ordered digoxin immune Fab (Digifab). The patient asks the nurse why the medication is ordered. What is the nurse's best response? a. "It will increase your heart rate." b. "This drug helps to lower your potassium levels." c. "It helps to convert the irregular heart rhythm to a more normal rhythm." d. "This drug is an antidote to digoxin and will help to lower the blood levels."

ANS: D Digoxin immune Fab (Digifab) is the antidote for a severe digoxin overdose. It is given intravenously

When a patient is taking an anticholinergic such as benztropine (Cogentin) as part of the treatment for Parkinson's disease, the nurse should include which information in the teaching plan? a. Minimize the amount of fluid taken while on this drug. b. Discontinue the medication if adverse effects occur. c. Take the medication on an empty stomach to enhance absorption. d. Use artificial saliva, sugarless gum, or hard candy to counteract dry mouth.

ANS: D Dry mouth can be managed with artificial saliva through drops or gum, frequent mouth care, forced fluids, and sucking on sugar-free hard candy. Anticholinergics should be taken with or after meals to minimize GI upset and must not be discontinued suddenly. The patient must drink at least 3000 mL/day unless contraindicated. Drinking water is important, even if the patient is not thirsty or in need of hydration, to prevent and manage the adverse effect of constipation.

A 72-year-old man has a new prescription for an anticholinergic drug. He is an active man and enjoys outdoor activities, such as golfing and doing his own yard work. What will the nurse emphasize to him during the teaching session about his drug therapy? a. Drowsiness may interfere with his outdoor activities. b. Increased salivation may occur during exercise and outside activities. c. Fluid volume deficits may occur as a result of an increased incidence of diarrhea. d. He will need to take measures to reduce the occurrence of heat stroke during his activities.

ANS: D Elderly patients who take an anticholinergic drug need to be reminded that they are at a greater risk for suffering heat stroke because of decreased sweating and loss of normal heat-regulating mechanisms.

The nurse recognizes that use of estrogen drugs is contraindicated in which patient? a. A patient who has atrophic vaginitis b. A patient who has inoperable prostate cancer c. A woman who has just given birth and wants to prevent postpartum lactation d. A woman with a history of thrombophlebitis

ANS: D Estrogenic drugs are contraindicated in people who have active thromboembolic disorders and in those with histories of thromboembolic disease. Atrophic vaginitis and inoperable prostate cancer are potential indications for estrogen therapy. Estrogen is not used to prevent lactation.

The nurse is reviewing instructions for vaginal antifungal drugs with a patient. Which statement by the nurse is an appropriate instruction regarding these drugs? a. "The medication can be stopped when your symptoms are relieved." b. "Discontinue this medication if menstruation begins." c. "Daily douching is part of the treatment for vaginal fungal infections." d. "Abstain from sexual intercourse until the treatment has been completed and the infection has resolved."

ANS: D Female patients taking antifungal medications for the treatment of vaginal infections need to abstain from sexual intercourse until the treatment has been completed and the infection has resolved. The medication needs to be taken for as long as prescribed. Instruct patients to continue to take the medication even if they are actively menstruating. Douching is not an appropriate intervention.

A patient has been taking the monoamine oxidase inhibitor (MAOI) phenelzine (Nardil) for 6 months. The patient wants to go to a party and asks the nurse, "Will just one beer be a problem?" Which advice from the nurse is correct? a. "You can drink beer as long as you have a designated driver." b. "Now that you've had the last dose of that medication, there will be no further dietary restrictions." c. "If you begin to experience a throbbing headache, rapid pulse, or nausea, you'll need to stop drinking." d. "You need to avoid all foods that contain tyramine, including beer, while taking this medication."

ANS: D Foods containing tyramine, such as beer and aged cheeses, should be avoided while a patient is taking an MAOI. Drinking beer while taking an MAOI may precipitate a dangerous hypertensive crisis.term-153

A 49-year-old patient is in the clinic for a follow-up visit 6 months after starting a beta blocker for treatment of hypertension. During this visit, his blood pressure is 169/98 mm Hg, and he eventually confesses that he stopped taking this medicine 2 months ago because of an "embarrassing problem." What problem did the patient most likely experience with this medication that caused him to stop taking it? a. Urge incontinence b. Dizziness when standing up c. Excessive flatus d. Impotence

ANS: D Impotence is a potential adverse effect of beta blockers and may cause patients to stop taking the medication. The other options are not adverse effects of beta blockers.

The nurse is administering a vancomycin (Vancocin) infusion. Which measure is appropriate for the nurse to implement in order to reduce complications that may occur with this drug's administration? a. Monitoring blood pressure for hypertension during the infusion b. Discontinuing the drug immediately if red man syndrome occurs c. Restricting fluids during vancomycin therapy d. Infusing the drug over at least 1 hour

ANS: D Infuse the medication over at least 1 hour to reduce the occurrence of red man syndrome. Adequate hydration (at least 2 L of fluid in 24 hours) during vancomycin therapy is important for the prevention of nephrotoxicity. Hypotension may occur during the infusion, especially if it is given too rapidly

A woman who is planning to become pregnant should ensure that she receives adequate levels of which supplement to reduce the risk for fetal neural tube defects? a. Vitamin B12 b. Vitamin D c. Iron d. Folic acid

ANS: D It is recommended that administration of folic acid be begun at least 1 month before pregnancy and continue through early pregnancy to reduce the risk for fetal neural tube defects.

The nurse is reviewing new medication orders for a patient who has an epidural catheter for pain relief. One of the orders is for enoxaparin (Lovenox), a low-molecular-weight heparin (LMWH). What is the nurse's priority action? a. Give the LMWH as ordered. b. Double-check the LMWH order with another nurse, and then administer as ordered. c. Stop the epidural pain medication, and then administer the LMWH. d. Contact the prescriber because the LMWH cannot be given if the patient has an epidural catheter.

ANS: D LMWHs are contraindicated in patients with an indwelling epidural catheter; they can be given 2 hours after the epidural is removed. This is very important to remember, because giving an LMWH with an epidural has been associated with epidural hematoma

The nurse is teaching a patient who will be taking a proton pump inhibitor as long-term therapy about potential adverse effects. Which statement is correct? a. Proton pump inhibitors can cause diarrhea. b. These drugs can cause nausea and anorexia. c. Proton pump inhibitors cause drowsiness. d. Long-term use of these drugs may contribute to osteoporosis.

ANS: D New concerns have arisen over the potential for long-term users of proton pump inhibitors (PPIs) to develop osteoporosis. This is thought to be due to the inhibition of stomach acid, and it is speculated that PPIs speed up bone mineral loss.

The nurse is screening a patient who will be taking a nonspecific/nonselective beta blocker. Which condition, if present, may cause serious problems if the patient takes this medication? a. Angina b. Hypertension c. Glaucoma d. Asthma

ANS: D Nonspecific/nonselective beta-blocking drugs may precipitate bradycardia, hypotension, heart block, heart failure, bronchoconstriction, and/or increased airway resistance. Therefore, any preexisting respiratory conditions such as asthma might be worsened by the concurrent use of any of these medications

A patient is taking omeprazole (Prilosec) for the treatment of gastroesophageal reflux disease (GERD). The nurse will include which statement in the teaching plan about this medication? a. "Take this medication once a day after breakfast." b. "You will be on this medication for only 2 weeks for treatment of the reflux disease." c. "The medication may be dissolved in a liquid for better absorption." d. "The entire capsule must be taken whole, not crushed, chewed, or opened."

ANS: D Omeprazole needs to be taken before meals, and an entire capsule must be taken whole, not crushed, chewed, opened, or dissolved in liquid when treating GERD. This medication is used on a long-term basis to maintain healing.

A patient with hypothyroidism is given a prescription for levothyroxine (Synthroid). When the nurse explains that this is a synthetic form of the thyroid hormone, he states that he prefers to receive more "natural" forms of drugs. What will the nurse explain to him about the advantages of levothyroxine? a. It has a stronger effect than the natural forms. b. Levothyroxine is less expensive than the natural forms. c. The synthetic form has fewer adverse effects on the gastrointestinal tract. d. The half-life of levothyroxine is long enough to permit once-daily dosing.

ANS: D One advantage of levothyroxine over the natural forms is that it can be administered only once a day because of its long half-life

The nurse is teaching a review class to nurses about diabetes mellitus. Which statement by the nurse is correct? a. "Patients with type 2 diabetes will never need insulin." b. "Oral antidiabetic drugs are safe for use during pregnancy." c. "Pediatric patients cannot take insulin." d. "Insulin therapy is possible during pregnancy if managed carefully."

ANS: D Oral medications are generally not recommended for pregnant patients because of a lack of firm safety data. For this reason, insulin therapy is the only currently recommended drug therapy for pregnant women with diabetes. Insulin is given to pediatric patients, with extreme care. Patients with type 2 diabetes may require insulin in certain situations or as their disease progresses.

The nurse is teaching a patient with iron-deficiency anemia about foods to increase iron intake. Which food may enhance the absorption of oral iron forms? a. Milk b. Yogurt c. Antacids d. Orange juice

ANS: D Orange juice contains ascorbic acid, which enhances the absorption of oral iron forms; antacids, milk, and yogurt may interfere with absorption.

Furosemide (Lasix) is prescribed for a patient who is about to be discharged, and the nurse provides instructions to the patient about the medication. Which statement by the nurse is correct? a. "Take this medication in the evening." b. "Avoid foods high in potassium, such as bananas, oranges, fresh vegetables, and dates." c. "If you experience weight gain, such as 5 pounds or more per week, be sure to tell your physician during your next routine visit." d. "Be sure to change positions slowly and rise slowly after sitting or lying so as to prevent dizziness and possible fainting because of blood pressure changes."

ANS: D Orthostatic hypotension is a possible problem with diuretic therapy. Foods high in potassium should be eaten more often, and the drug needs to be taken in the morning so that the diuretic effects do not interfere with sleep. A weight gain of 5 pounds or more per week must be reported immediately.

The nurse is discussing adverse effects of antitubercular drugs with a patient who has active tuberculosis. Which potential adverse effect of antitubercular drug therapy should the patient report to the prescriber? a. Gastrointestinal upset b. Headache and nervousness c. Reddish-orange urine and stool d. Numbness and tingling of extremities

ANS: D Patients on antitubercular therapy should report experiencing numbness and tingling of extremities, which may indicate peripheral neuropathy. Some drugs may color the urine, stool, and other body secretions reddish-orange, but this is not an effect that needs to be reported. Patients need to be informed of this expected effect.

The nurse is teaching patients about self-injection of insulin. Which statement is true regarding injection sites? a. Avoid the abdomen because absorption there is irregular. b. Choose a different site at random for each injection. c. Give the injection in the same area each time. d. Rotate sites within the same location for about 1 week before rotating to a new location.

ANS: D Patients taking insulin injections need to be instructed to rotate sites, but to do so within the same location for about 1 week (so that all injections are rotated in one area—for example, the right arm—before rotating to a new location, such as the left arm). Also, each injection needs to be at least to 1 inch away from the previous site.

A patient with the diagnosis of schizophrenia is hospitalized and is taking a phenothiazine drug. Which statement by this patient indicates that he is experiencing a common adverse effect of phenothiazines? a. "I can't sleep at night." b. "I feel hungry all the time." c. "Look at how red my hands are." d. "My mouth has been so dry lately."

ANS: D Phenothiazines produce anticholinergic-like adverse effects of dry mouth, urinary hesitancy, and constipation.

A woman visits a health center requesting oral contraceptives. Which laboratory test is most important for the nurse to assess before the patient begins oral contraceptive therapy? a. Complete blood count b. Serum potassium level c. Vaginal cultures d. Pregnancy test

ANS: D Pregnancy should be ruled out before beginning oral contraceptive therapy because the medications can be harmful to the fetus; they are classified as pregnancy category X.

A patient who started taking orlistat (Xenical) 1 month ago calls the clinic to report some "embarrassing" adverse effects. She tells the nurse that she has had episodes of "not being able to control my bowel movements." Which statement is true about this situation? a. These are expected adverse effects that will eventually diminish. b. The patient will need to stop this drug immediately if these adverse effects are occurring. c. The patient will need to increase her fat intake to prevent these adverse effects. d. The patient will need to restrict fat intake to less than 30% to help reduce these adverse effects.

ANS: D Restricting dietary intake of fat to less than 30% of total calories can help reduce some of the GI adverse effects, which include oily spotting, flatulence, and fecal incontinence.

A patient newly diagnosed with tuberculosis (TB) has been taking antitubercular drugs for 1 week calls the clinic and is very upset. He says, "My urine is dark orange! What's wrong with me?" Which response by the nurse is correct? a. "You will need to stop the medication, and it will go away." b. "It's possible that the TB is worse. Please come in to the clinic to be checked." c. "This is not what we usually see with these drugs. Please come in to the clinic to be checked." d. "This is an expected side effect of the medicine. Let's review what to expect."

ANS: D Rifampin, one of the first-line drugs for TB, causes a red-orange-brown discoloration of urine, tears, sweat, and sputum. Patients need to be warned about this side effect. The other options are incorrect.

A 51-year-old woman will be taking selective estrogen receptor modulators (SERMs) as part of treatment for postmenopausal osteoporosis. The nurse reviews potential contraindications, including which condition? a. Hypocalcemia b. Breast cancer c. Stress fractures d. Venous thromboembolism

ANS: D SERMs such as raloxifene are contraindicated in women with a venous thromboembolic disorder, including deep vein thrombosis, pulmonary embolism, or a history of such disorders.

A patient is complaining of excessive and painful gas. The nurse checks the patient's medication orders and prepares to administer which drug for this problem? a. Famotidine (Pepcid) b. Aluminum hydroxide and magnesium hydroxide (Maalox or Mylanta) c. Calcium carbonate (Tums) d. Simethicone (Mylicon)

ANS: D Simethicone alters the elasticity of mucus-coated bubbles, causing them to break, and is an overthe-counter antiflatulent.

An 18-year-old basketball player fell and twisted his ankle during a game. The nurse will expect to administer which type of analgesic? a. Synthetic opioid, such as meperidine (Demerol) b. Opium alkaloid, such as morphine sulfate c. Opioid antagonist, such as naloxone HCL (Narcan) d . Nonopioid analgesic, such as indomethacin (Indocin)

ANS: D Somatic pain, which originates from skeletal muscles, ligaments, and joints, usually responds to nonopioid analgesics such as nonsteroidal anti-inflammatory drugs (NSAIDs). The other options are not the best choices for somatic pain.

The nurse will monitor a patient for signs and symptoms of hyperkalemia if the patient is taking which of these diuretics? a. Hydrochlorothiazide (HydroDIURIL) b. Furosemide (Lasix) c. Acetazolamide (Diamox) d. Spironolactone (Aldactone)

ANS: D Spironolactone (Aldactone) is a potassium-sparing diuretic, and patients taking this drug must be monitored for signs of hyperkalemia. The other drugs do not cause hyperkalemia but instead cause hypokalemia.

When teaching a patient who is starting metformin (Glucophage), which instruction by the nurse is correct? a. "Take metformin if your blood glucose level is above 150 mg/dL." b. "Take this 60 minutes after breakfast." c. "Take the medication on an empty stomach 1 hour before meals." d. "Take the medication with food to reduce gastrointestinal (GI) effects."

ANS: D The GI adverse effects of metformin can be reduced by administering it with meals.

A patient has been taking the selective serotonin reuptake inhibitor (SSRI) sertraline (Zoloft) for about 6 months. At a recent visit, she tells the nurse that she has been interested in herbal therapies and wants to start taking St. John's wort. Which response by the nurse is appropriate? a. "That should be no problem." b. "Good idea! Hopefully you'll be able to stop taking the Zoloft." c. "Be sure to stop taking the herb if you notice a change in side effects." d. "Taking St. John's wort with Zoloft may cause severe interactions and is not recommended."

ANS: D The herbal product St. John's wort must not be used with SSRIs. Potential interactions include confusion, agitation, muscle spasms, twitching, and tremors.

A patient has an order for cyclosporine (Sandimmune). The nurse finds that cyclosporinemodified (Neoral) is available in the automated medication cabinet. Which action by the nurse is correct? a. Hold the dose until the prescriber makes rounds. b. Give the cyclosporine-modified drug. c. Double-check the order, and then give the cyclosporine-modified drug. d. Notify the pharmacy to obtain the Sandimmune form of the drug.

ANS: D The nurse must double-check the formulation before giving cyclosporine. Cyclosporine-modified products (such as Neoral or Gengraf) are interchangeable with each other but are not interchangeable with Sandimmune. In this case, the nurse must obtain the Sandimmune form of the drug from the pharmacy.

During a routine appointment, a patient with a history of seizures is found to have a phenytoin (Dilantin) level of 23 mcg/mL. What concern will the nurse have, if any? a. The patient is at risk for seizures because the drug level is not at a therapeutic level. b. The patient's seizures should be under control because this is a therapeutic drug level. c. The patient's seizures should be under control if she is also taking a second antiepileptic drug. d. The drug level is at a toxic level, and the dosage needs to be reduced.

ANS: D Therapeutic drug levels for phenytoin are usually 10 to 20 mcg/mL (see Table 14-6).

A patient has been treated with antiparkinson medications for 3 months. What therapeutic responses should the nurse look for when assessing this patient? a. Decreased appetite b. Gradual development of cogwheel rigidity c. Newly developed dyskinesias d. Improved ability to perform activities of daily living

ANS: D Therapeutic responses to antiparkinson agents include an improved sense of well-being, improved mental status, increased appetite, increased ability to perform activities of daily living and to concentrate and think clearly, and less intense parkinsonian manifestations.

A 19-year-old woman has been diagnosed with hypothyroidism and has started thyroid replacement therapy with levothyroxine (Synthroid). After 6 months, she calls the nurse to say that she feels better and wants to stop the medication. Which response by the nurse is correct? a. "You can stop the medication if your symptoms have improved." b. "You need to stay on the medication for at least 1 year before a decision about stopping it can be made." c. "You need to stay on this medication until you become pregnant." d. "Medication therapy for hypothyroidism is usually lifelong, and you should not stop taking the medication."

ANS: D These medications must never be abruptly discontinued, and lifelong therapy is usually the norm.

A patient has been prescribed warfarin (Coumadin) in addition to a heparin infusion. The patient asks the nurse why he has to be on two medications. The nurse's response is based on which rationale? a. The oral and injection forms work synergistically. b. The combination of heparin and an oral anticoagulant results in fewer adverse effects than heparin used alone. c. Oral anticoagulants are used to reach an adequate level of anticoagulation when heparin alone is unable to do so. d. Heparin is used to start anticoagulation so as to allow time for the blood levels of warfarin to reach adequate levels.

ANS: D This overlap therapy is required in patients who have been receiving heparin for anticoagulation and are to be switched to warfarin so that prevention of clotting is continuous. This overlapping is done purposefully to allow time for the blood levels of warfarin to rise, so that when the heparin is eventually discontinued, therapeutic anticoagulation levels of warfarin will have been achieved. Recommendations are to continue overlap therapy of the heparin and warfarin for at least 5 days; the heparin is stopped after day 5 when the international normalized ratio (INR) is above 2.

A patient on diuretic therapy calls the clinic because he's had the flu, with "terrible vomiting and diarrhea," and he has not kept anything down for 2 days. He feels weak and extremely tired. Which statement by the nurse is correct? a. "It's important to try to stay on your prescribed medication. Try to take it with sips of water." b. "Stop taking the diuretic for a few days, and then restart it when you feel better." c. "You will need an increased dosage of the diuretic because of your illness. Let me speak to the physician." d. "Please come into the clinic for an evaluation to make sure there are no complications."

ANS: D Vomiting and diarrhea cause fluid and electrolyte loss. The patient must not continue to take the diuretic until these problems have stopped. He needs to be checked for possible hypokalemia and dehydration.

When assessing a patient who is receiving a loop diuretic, the nurse looks for the manifestations of potassium deficiency, which would include what symptoms? (Select all that apply.) a. Dyspnea b. Constipation c. Tinnitus d. Muscle weakness e. Anorexia f. Lethargy

ANS: D, E, F d. Muscle weakness e. Anorexia f. Lethargy Symptoms of hypokalemia include anorexia, nausea, lethargy, muscle weakness, mental confusion, and hypotension.

Opioid Contraindications

Acute bronchial asthma, emphysema, increased intracranial pressure

Allopurinol (Zyloprim)

Anti-gout med Used to prevent uric acid production

Why is Benztropine used?

Anticholinergic drug used for Parkinsons Disease and extrapyramidal symptoms from antipsychotic drugs - Caution during hot weather or exercise because it may cause hyperthermia

Digoxin patient assessment

Assess: BP Apical pulse for 1 minute Weight, I&O measures

Adverse effects of carbamazepine

Autoinduction of hepatic enzymes

what are some adverse effects of benztropine?

Avoid alcohol!!! - dry throat - tachycardia - confusion - disorientation - toxic psychosis - urinary retention - constipation - nausea, and vomiting

MAOIs food interactions

Avoid foods high in tyramine ( they increase blood pressure) These include smoked/aged meats, pepperoni aged cheeses, beer, red wine, yeast extracts, italian broad beans etc

The nurse is providing patient teaching for a patient who is starting antitubercular drug therapy. Which of these statements should be included? (Select all that apply.) a. "Take the medications until the symptoms disappear." b. "Take the medications at the same time every day." c. "You will be considered contagious during most of the illness and must take precautions to avoid spreading the disease." d. "Stop taking the medications if you have severe adverse effects." e. "Avoid alcoholic beverages while on this therapy." f. "If you notice reddish-brown or reddish-orange urine, stop taking the drug and contact your doctor right away." g. "If you experience a burning or tingling in your fingers or toes, report it to your prescriber immediately." h. "Oral contraceptives may not work while you are taking these drugs, so you will have to use another form of birth control."

B, E, G, H b. "Take the medications at the same time every day." e. "Avoid alcoholic beverages while on this therapy." g. "If you experience a burning or tingling in your fingers or toes, report it to your prescriber immediately." h. "Oral contraceptives may not work while you are taking these drugs, so you will have to use another form of birth control." Medications for tuberculosis must be taken on a consistent schedule to maintain blood levels. Medication therapy for tuberculosis may last up to 24 months, long after symptoms disappear, and patients are infectious during the early part of the treatment. Compliance with antitubercular drug therapy is key, so if symptoms become severe, the prescriber should be contacted for an adjustment of the drug therapy. The medication must not be stopped. Because of potential liver toxicity, patients on this drug therapy must not drink alcohol. Discoloration of the urine is an expected adverse effect, and patients need to be warned about it beforehand. Burning or tingling in the fingers or toes may indicate that peripheral neuropathy is developing, and the prescriber needs to be notified immediately. A second form of birth control must be used because antitubercular drug therapy makes oral contraceptives ineffective.

What do we give for alcohol withdrawal?

Benzodiazepines

Metformin (Glucophage)

Biguanide - Monitor kidney function (increased risk of lactic acidosis) - Give with food to minimize GI upset - will need to be discontinued if the patient is to undergo studies with contrast dye because of possible renal effects—check with the prescriber

Ganciclovir adverse effect

Bone marrow toxicity

what type of med is Modafinil?

CNS Stimulant

Bupropion patient teaching

Cannot take if history of seizures

What is methotrexate used for?

Chemotherapy agent & rheumatoid arthritis

Ciprofloxacin contraindications

Children under 18 years

HAART (highly active antiretroviral therapy)

Combination of drugs effective against AIDS - Consists of 3 drugs - these meds work in different ways to reduce the viral load

Penicillin hypersensitivity reactions

Cross-sensitivity with cephalosporins

Metoclopramide can cause :

EPS and tardive dyskinesia

It is ____________ to obtain cultures from appropriate sites ___________ beginning antibiotic therapy.

ESSENTIAL; BEFORE

what is used for life-threatening bleeding from warfarin

Fresh Frozen Plasma

What is the most commonly used loop diuretic?

Furosemide (Lasix)

Which drug is used as one time dose for vaginal yeast infections?

Fluconazole

Benzodiazepines antidote

Flumazenil

Potassium sparing diuretics adverse reaction

Gynecomastia

Thiazide diuretics indications

Hypertension (one of the most prescribed group of drugs for this)

What time should MAOIs be taken?

In the morning because they can cause insomnia

What is Oseltamivir (tamiflu) used for?

Influenza A and B Reduces duration of illness

How long is antiseizure therapy?

It is usually lifelong

What is the approach to estrogen replacement?

LOWEST dose possible at first

Rapid acting insulin

Lispro (Humalog)

HMG-CoA Inhibitors (Statins) adverse effects

Liver failure Rhabdomylosis

Digoxin adverse effects

Low potassium levels increase its toxicity - Colored vision (seeing green, yellow, purple), halo vision, flickering lights - Dysrhythmias- life threatening ventricular dysrhymias - confusion - Anorexia, nausea, vomiting, diarrhea

Cyclobenzaprine (Flexeril)

Muscle Relaxant

Intermediate acting insulin

NPH

Opioid reversal

Naloxone (Narcan)

epoetin alfa stimulates production of ____________.

RBCs

What happens if Vancomycin is infused in under 60 minutes?

Red man syndrome - Flushing or itching of head, neck, face, upper trunk, decrease in blood pressure, back pain, fever, chills, paresthesias, tinnitis - Antihistamine may be ordered to reduce these effects

short acting insulin

Regular insulin

Ciprofloxacin adverse effects

Ruptured tendons,* tendonitis,*

Patient teaching for Orlistat

SE - diarrhea, flatus, oily stools, fecal incontinence Considerations - may have problem with malabsorption, take daily multivitamin. Caution in patients with renal insufficiency, liver disease, or gallbladder disease. Do not administer with cyclosporin. Should be taken with foods and with some fats but not too much because it will cause fecal incontinentence

What should you look out for when administering Methotrexate?

Severe reactions (give Leucovorin to decrease)

antiepileptic drugs adverse effects

Sore throat, fever (blood dyscrasias may occur with hydantoins)

Potassium sparing diuretics

Spironolactone (Aldactone)

What herb has a severe interactions if taken with MAOIs and SSRIs?

St. Johns wort

HMG-CoA Inhibitors

Statin - Given in evening or at bedtime- more effective results - Watch liver function tests

penicillin patient teaching

Take oral doses with water (not juices) because acidic fluids may nullify the drug's antibacterial action.

What alpha blocker is primarily used to treat BPH?

Tamsulosin

Contraceptive drugs adverse effects

Thromboembolism possible PE MI stroke Hypertension

Warfarin antidote

Vitamin K

What are EPS treated with?

benztropine

Cephalosporins contraindications

allergy to penicillin

digoxin reversal

digoxin immune FAB

What is the drug of choice for the treatment of many severe systemic fungal infections?

amphotericin B

What does Ciprofloxacin treat?

anthrax

MAOIs

antidepressants Interacts with most medications

Pyrazinamide adverse reaction

antitubercular med adverse reaction: - hepatotoxicity

Rifampin adverse reactions

antitubercular med adverse reaction: - reddish-orange discoloration of body fluids

Isoniazid adverse reactions

antitubercular med adverse reactions: - Hepato and neuro toxicity (peripheral neuropathy) - jaundice, can cause pyridoxine (B6 deficiency)

carbamazepine food interactions

avoid grapefruit

What should be used while on TB therapy?

barrier contraceptives

Excessive consumption of ___________ can lead to additive hypo_________ in clients taking thiazides

black licorice; kalemia

Clozapine adverse effect

blood dyscrasias

What is the most common adverse effect of levothyrozine?

cardiac dysrhythmias

What is the most serious adverse reaction of lithium?

cardiac dysrhythmias

Midazolam (Versed) Indications

class: Benzodiazepine Causes amnesia and anxiolysis (reduced anxiety) as well as sedation.

cyclosporine (Sandimmune)

class: immunosuppresants, antirheumatics (DMARD) Indication: prevention of rejection in transplantation, treatment of severe RA, management of ulcerative colitis Nursing Considerations: -may cause seizures, tremors, hypertension, hepatotoxicity, diarrhea, N/V, gingival hyperplasia - increases immune suppression with corticosteroids - avoid grapefruit juice while taking this medications - assess for signs of organ rejection - monitor renal panel, liver enzymes - take medication as directed - lifelong therapy required for transplant patients - instruct pt on how to take blood pressure

What should be assessed before administering a statin?

creatinine kinase levels

Long acting insulin

glargine (Lantus)

Digoxin (Lanoxin)

heart failure drug

Heparin adverse effects

hematuria, melena (blood in the stool), petechiae, ecchymoses, and gum or mucous membrane bleeding

What is acyclovir used to treat?

herpes viral infections

What does Modafinil treat?

improvement of wakefulness in clients with excessive daytime sleepiness associated with narcolepsy

Isoniazid food interactions

interacts with foods containing tyramine (aged cheese, caffeinated beverages, yogurt, chocolate, red wine, beer, and pepperoni)

when should levodopa-carbidopa be taken?

on an empty stomach

What is the most common adverse effect of MAOIs?

orthostatic hypotension

What does lithium treat?

mania

Morphine

may be used at end of life to increase aveolar gas exchange and decrease workload of breathing

Codeine

may be used to suppress cough

How long is TB therapy?

may last for up to 24 months

Phenytoin (Dilantin)

most commonly prescribed antiepileptic drug in the United States

The most common adverse effects of antibiotics are:

nausea, vomiting, and diarrhea

Aminoglycosides adverse effect

nephrotoxicity

Vancomycin adverse effects

ototoxicity and nephrotoxicity

MAOIs disadvantage

potential to cause hypertensive crisis when taken with tyramine

Heparin antidote

protamine sulfate

Antacid adverse effects

rebound acidity alkalosis

Beta Blockers

reduce blood pressure

What is prednisone used for?

steroid used to treat inflammation, allergy, asthma, RA, Cushings, decreases normal immune response

What does prednisone do?

suppresses the immune system and treats inflammation

Opioids

synthetic opiates that are prescribed for pain relief

Disulfiram

treatment for alcoholism -lifelong

What do we give Bethanechol for?

treatment of acute postoperative and postpartum non-obstructive urinary retention

What is vasopressin used for?

treatment of diabetes insipidus

Methadone

used to treat addictions

Hydromorphone

very strong (7x stronger than morphine) and may be used in PCA pump- itching is common and Benadryl often prescribed

Diuretics patient teaching

• Teach clients to maintain proper nutritional and fluid volume status • Teach clients to eat more potassium-rich foods when taking any but the potassium-sparing drugs • Foods high in potassium include bananas, oranges, dates, apricots, raisins, broccoli, green beans, potatoes, meats, fish, and legumes

PPD (purified protein derivative)

•A diagnostic injection given intradermally in doses of 5 tuberculin units (0.1 mL) to detect exposure to the TB organism •Positive result is indicated by induration (not erythema) at the site of injection

BCG (Bacille Calmette-Guérin)

•A vaccine injection derived from an inactivated strain of Mycobacterium bovis •Can cause false-positive results on the tuberculin skin test


Set pelajaran terkait

Business Policy - Chapter 7: Innovation and Entrepreneurship

View Set

Lecture 3: Ocean Bathymetry and Seafloor Topography (i.e., How deep is the ocean and what does the seafloor look like?

View Set

Hip Hop Culture Chapter 3 Study Questions

View Set

BUS 300: Chapters 15, 16, 17, & 18

View Set

US History Semester 2 Final Exam Study Guide

View Set

Unit 2 AP Bio - The Central Dogma, Enzymes, Gene Expression, and Genetic Engineering

View Set

PN Pharmacology Online Practice 2023 B

View Set

Assignment 8 - Steps in the Underwriting Process

View Set